Минус косинус тангенс как найти

минус в аргументе

Примеры: 

(sin⁡(-π)=-sin,⁡π)
(cos⁡(-225^° )=cos⁡, 225^°)
(tg(-frac{π}{2}-x)=-tg(frac{π}{2}+x))

И сразу два важных замечания.

Замечание №1

Многие ученики думают, что если можно вынести минус из тригонометрической функции, то можно вынести и число, но это не так:

(sin⁡,2x≠2 sin,⁡x)
(cos⁡,3x≠3cos,⁡x)
(tg,4x≠4tg,x)

Замечание №2

(sin^2⁡(-x)=sin^2⁡x)
(cos^2⁡(-x)=cos^2⁡x)
(tg^2 (-x)=tg^2 x)
(ctg^2 (-x)=ctg^2 x)

Квадрат меняет ситуацию. Всё дело в том, что (sin^2⁡(-x)=(sin⁡(-x) )^2=(-sin,⁡x )^2=sin^2⁡x), т.е. минус все равно выносится, но так как синуса два и они перемножаются, то в итоге получается плюс.

Примеры из ЕГЭ

Пример (ЕГЭ). Найдите значение выражения (24sqrt{2},cos⁡(-frac{π}{3}),sin⁡(-frac{π}{4})).
Решение. (24sqrt{2},cos⁡(-frac{π}{3}),sin⁡(-frac{π}{4})=-24sqrt{2},cos⁡frac{π}{3},sin⁡frac{π}{4}).

косинус пи на 3, синус пи на4

Из рисунка видно, что и косинус, и синус положителен. Косинус из трех стандартных значений (frac{1}{2}), (frac{sqrt{2}}{2}), (frac{sqrt{3}}{2}) принимает наименьшее т.е. (cos,⁡frac{π}{3}=frac{1}{2}). Синус из трех стандартных значений будет равен среднему т.е. (sin⁡,frac{π}{4}=frac{sqrt{2}}{2}). Получается:

(-24sqrt{2},cos⁡frac{π}{3},sin⁡frac{π}{4}=-24sqrt{2}cdot)(frac{1}{2})(cdot)(frac{sqrt{2}}{2})(=)(frac{-24sqrt{2}cdotsqrt{2}}{4})(=)(frac{-24cdot 2}{4})(=-6cdot2=-12)

Ответ: (-12).

Если вы не поняли почему (frac{π}{3}) и (frac{π}{4}) находятся на круге там, где мы из обозначили, то читайте статью «Как обозначать числа с пи на числовой окружности?». А если не поняли, как мы нашли синус и косинус, то читайте статью «Как найти синус и косинус без тригонометрической таблицы».

Пример (ЕГЭ). Найдите значение выражения (44sqrt{3},tg,(-480^° )).
Решение. (44sqrt{3},tg(-480^° )=-44sqrt{3},tg(480^° )=-44sqrt{3},tg(360^°+120^° )=-44sqrt{3},tg(360^°+90^°+30^°)).

Находим (480^°) на окружности:

тангенс 480 градусов

Соединяем точку, соответствующую (480^°) и центр окружности, и продляем до оси тангенсов:

тангенс 480 градусов

Мы попадаем в самое маленькое (из стандартных) значение тангенса.
Значит, (tg(480^° )=-sqrt{3}).
В итоге имеем: (44sqrt{3} tg(-480^° )=-44sqrt{3}cdot(-sqrt{3})=44cdot 3=132).
Ответ: (132).

Если вам не понятно, как мы нашли значение тангенса, то читайте статью «Как найти тангенс и котангенс без тригонометрической таблицы?».

Доказательства формул с минусом в аргументе:

Для удобства сразу же приведем таблицу с всеми тригонометрическими тождествами. Всегда удобно открыть формулы в одном месте, выбрать нужную и решить пример. После таблицы мы по отдельности рассмотрим каждую тригонометрическую формулу: обсудим ее вывод и порешаем примеры.

  1. Основное тригонометрическое тождество:
    $$sin(alpha)^2+cos(alpha)^2=1;$$
  2. Определение тангенса и котангенса через синус и косинус:
    $$tg(alpha)=frac{sin(alpha)}{cos(alpha)};$$
    $$ctg(alpha)=frac{cos(alpha)}{sin(alpha)};$$
  3. Cвязь тангенса и котангенса:
    $$tg(alpha)=frac{1}{ctg(alpha)};$$
    $$tg(alpha)*ctg(alpha)=1;$$
  4. Тангенс через косинус. Котангенс через синус:
    $$tg(alpha)^2+1=frac{1}{cos(alpha)^2};$$
    $$ctg(alpha)^2+1=frac{1}{sin(alpha)^2};$$
  5. Синус суммы и разности:
    $$sin(alpha+beta)=sin(alpha)*cos(beta)+sin(beta)*cos(alpha);$$
    $$sin(alpha-beta)=sin(alpha)*cos(beta)-sin(beta)*cos(alpha);$$
  6. Косинус суммы и разности:
    $$cos(alpha+beta)=cos(alpha)*cos(beta)-sin(beta)*sin(alpha);$$
    $$cos(alpha-beta)=cos(alpha)*cos(beta)+sin(beta)*sin(alpha);$$
  7. Тангенс суммы и разности:
    $$tg(alpha+beta)=frac{tg(alpha)+tg(beta)}{1-tg(alpha)*tg(beta)};$$
    $$tg(alpha-beta)=frac{tg(alpha)-tg(beta)}{1+tg(alpha)*tg(beta)};$$
  8. Котангенс суммы и разности:
    $$сtg(alpha+beta)=frac{-1+сtg(alpha)*ctg(beta)}{ctg(alpha)+ctg(beta)};$$
    $$сtg(alpha-beta)=frac{-1-сtg(alpha)*ctg(beta)}{ctg(alpha)-ctg(beta)};$$
  9. Двойной угол:
    $$cos(2*alpha)=cos(alpha)^2-sin(alpha)^2=1-2*sin(alpha)^2=2*cos(alpha)^2-1;$$
    $$sin(2*alpha)=2*sin(alpha)*cos(alpha);$$
    $$tg(2*alpha)=frac{2*tg(alpha)}{1-tg(alpha)^2};$$
    $$ctg(2*alpha)=frac{ctg(alpha)^2-1}{2*ctg(alpha)};$$
  10. Тройной угол:
    $$cos(3*alpha)=cos(alpha)^3-3*sin(alpha)^2*cos(alpha)=-3*cos(alpha)+4*cos(alpha)^3;$$
    $$sin(3*alpha)=3*sin(alpha)*cos(alpha)^2-sin(alpha)^3=3*sin(alpha)-4*sin(alpha)^3;$$
    $$tg(3*alpha)=frac{3*tg(alpha)-tg(alpha)^3}{1-3*tg(alpha)^2};$$
    $$ctg(3*alpha)=frac{ctg(alpha)^3-3*ctg(alpha)}{3*ctg(alpha)^2-1};$$
  11. Формулы половинного угла:
    $$sin(frac{alpha}{2})^2=frac{1-cos(alpha)}{2};$$
    $$cos(frac{alpha}{2})^2=frac{1+cos(alpha)}{2};$$
    $$tg(frac{alpha}{2})^2=frac{1-cos(alpha)}{1+cos(alpha)};$$
    $$ctg(frac{alpha}{2})^2=frac{1+cos(alpha)}{1-cos(alpha)};$$
  12. Понижение степени:
    $$sin(alpha)^2=frac{1-cos(2*alpha)}{2};$$
    $$cos(alpha)^2=frac{1+cos(2*alpha)}{2};$$
    $$sin(alpha)^3=frac{3*sin(alpha)-sin(3*alpha)}{4};$$
    $$cos(alpha)^3=frac{3*cos(alpha)+cos(3*alpha)}{4};$$
    $$sin(alpha)^4=frac{3-4*cos(2*alpha)+cos(4*alpha)}{8};$$
    $$cos(alpha)^4=frac{3+4*cos(2*alpha)+cos(4*alpha)}{8};$$
  13. Преобразование суммы и разности тригонометрических функций:
    $$sin(alpha)+sin(beta)=2*sinleft(frac{alpha+beta}{2}right)*cosleft(frac{alpha-beta}{2}right);$$
    $$sin(alpha)-sin(beta)=2*sinleft(frac{alpha-beta}{2}right)*cosleft(frac{alpha+beta}{2}right);$$
    $$cos(alpha)+cos(beta)=2*cosleft(frac{alpha+beta}{2}right)*cosleft(frac{alpha-beta}{2}right);$$
    $$cos(alpha)-cos(beta)=-2*sinleft(frac{alpha+beta}{2}right)*sinleft(frac{alpha-beta}{2}right);$$
    $$cos(alpha)-cos(beta)=2*sinleft(frac{alpha+beta}{2}right)*sinleft(frac{beta-alpha}{2}right);$$
    $$tg(alpha)+tg(beta)=frac{sin(alpha+beta)}{cos(alpha)*cos(beta)};$$
    $$tg(alpha)-tg(beta)=frac{sin(alpha-beta)}{cos(alpha)*cos(beta)};$$
    $$ctg(alpha)+ctg(beta)=frac{sin(alpha+beta)}{sin(alpha)*sin(beta)};$$
    $$ctg(alpha)-ctg(beta)=frac{sin(beta-alpha)}{sin(alpha)*sin(beta)};$$
  14. Преобразование произведения тригонометрических функций:
    $$sin(alpha)*sin(beta)=frac{1}{2}*left(cos(alpha-beta)-cos(alpha+beta)right);$$
    $$cos(alpha)*cos(beta)=frac{1}{2}*left(cos(alpha-beta)+cos(alpha+beta)right);$$
    $$sin(alpha)*cos(beta)=frac{1}{2}*left(sin(alpha-beta)+sin(alpha+beta)right);$$
  15. Формулы подстановки тангенса:
    $$sin(alpha)=frac{2*tg(frac{alpha}{2})}{1+tg(frac{alpha}{2})^2};$$
    $$cos(alpha)=frac{1-tg(frac{alpha}{2})^2}{1+tg(frac{alpha}{2})^2};$$
    $$tg(alpha)=frac{2*tg(frac{alpha}{2})}{1-tg(frac{alpha}{2})^2};$$
    $$ctg(alpha)=frac{1-tg(frac{alpha}{2})^2}{2*tg(frac{alpha}{2})};$$
  16. Формулы приведения можно найти в отдельной статье

Зачем нужны тригонометрические формулы?

Как видите, тригонометрических формул очень много. Тут еще и не все приведены. Но на ваше счастье, учить всю эту таблицу не нужно. Достаточно знать только основные: №1-6, 9. Остальные на ЕГЭ по профильной математике встречаются крайне редко, а если и попадутся, то, скорее всего, будут даны в справочных материалах.

Но для участия в олимпиадах или, если вы хотите поступать в сильный математический ВУЗ через вступительные экзамены, то вам может понадобиться вся таблица. По крайней мере, у вас точно должно быть представление о существовании таких формул, чтобы их вывести в случае необходимости. Да, большинство из них легко выводятся.

Тригонометрические формулы нужны, чтобы связать все тригонометрические функции между собой. Если вы знаете одну из функций, например, синус, то, используя эти формулы, можно легко найти оставшиеся три тригонометрические функции (косинус, тангенс и котангенс). Кроме этого тождества позволяют упростить выражение под тригонометрической функцией: например, выразить синус от двойного угла через комбинацию тригонометрических функций от одинарного угла, что бывает очень полезно при решении тригонометрических уравнений и неравенств.

Обсудим и порешаем примеры на все формулы из таблицы.

Основное тригонометрическое тождество

$$mathbf{sin(alpha)^2+cos(alpha)^2=1;}$$

Эту формулу можно считать главной и самой часто используемой в тригонометрии. Она выводится при помощи определения синуса и косинуса через прямоугольный треугольник и теоремы Пифагора. Не буду еще раз описывать вывод, с ним можно познакомиться в самой первой главе по тригонометрии.

При помощи основного тригонометрического тождества очень удобно искать значение синуса, если известен косинус и наоборот. Разберем пример:

Пример 1
Найдите (3sqrt{2}*sin(alpha)=?), если (cos(alpha)=frac{1}{3}) и (alphain(0;frac{pi}{2})). (ЕГЭ)

Чтобы найти значение выражения (3sqrt{2}*sin(alpha)) необходимо сначала найти значение синуса.

Формула, которая связывает и синус, и косинус – это основное тригонометрическое тождество:
$$sin(alpha)^2+cos(alpha)^2=1;$$
Просто подставим в нее известное значение косинуса
$$sin(alpha)^2+left(frac{1}{3}right)^2=1;$$
$$sin(alpha)^2+frac{1}{9}=1;$$
$$sin(alpha)^2=1-frac{1}{9};$$
$$sin(alpha)^2=frac{8}{9};$$
$$sin(alpha)=pmsqrt{frac{8}{9}}=pmfrac{2sqrt{2}}{3};$$
Обратите внимание на знак (pm), отрицательное значение синуса нас тоже устраивает, так как при подстановке и возведении в квадрат знак минус исчезает.

В задании указано, что это пример из ЕГЭ первой части, значит должен быть только один ответ. Какое же значение синуса нам выбрать: положительное или отрицательное?

В этом нам поможет дополнительное условие на (alphain(0;frac{pi}{2})), что соответсвует первой четверти на тригонометрической окружности. Раз (alpha) лежит в первой четверти, то синус должен быть положительный. Выбираем положительное значение синуса:
$$sin(alpha)=frac{2sqrt{2}}{3};$$
И подставим найденное значение в искомое выражение:
$$3sqrt{2}*sin(alpha)=3sqrt{2}*frac{2sqrt{2}}{3}=4.$$

Ответ: (4.)

Аналогично по основному тригонометрическому тождеству можно находить значение косинуса, если известен синус.

Основные тригонометрическое тождество это ключ к решению более половины всех тригонометрических уравнений.

Основные связи тригонометрических функций

А как найти тангенс или котангенс, если нам, например, известен косинус? Посмотрите на формулы №2, для того, чтобы найти тангенс, нужно знать и косинус, и синус:

$$mathbf{tg(alpha)=frac{sin(alpha)}{cos(alpha)};}$$
$$mathbf{ctg(alpha)=frac{cos(alpha)}{sin(alpha)};}$$

Но зная косинус, мы легко можем найти синус по основному тригонометрическому тождеству, а потом уже найти тангенс.

Пример 2
Найдите (tg(alpha)) и (ctg(alpha)), если (cos(alpha)=frac{sqrt{10}}{10}) и (alpha in (frac{3pi}{2};2pi)).

Сначала находим значение синуса:
$$sin(alpha)^2+cos(alpha)^2=1;$$
$$sin(alpha)^2+left(frac{sqrt{10}}{10}right)^2=1;$$
$$sin(alpha)^2+frac{1}{10}=1;$$
$$sin(alpha)^2=1-frac{1}{10};$$
$$sin(alpha)^2=frac{9}{10};$$
$$sin(alpha)=pmsqrt{frac{9}{10}}=pmfrac{3}{sqrt{10}};$$
Так как по условию задачи (alpha in (frac{3pi}{2};2pi)), что соответсвует четвертой четверти на тригонометрической окружности, то (sin(alpha)<0). Выбираем отрицательное значение:
$$sin(alpha)=-frac{3}{sqrt{10}};$$
Теперь нам известны значения и косинуса, и синуса, можем найти тангенс:
$$tg(alpha)=frac{sin(alpha)}{cos(alpha)}=frac{-frac{3}{sqrt{10}}}{frac{sqrt{10}}{10}}=-frac{3}{sqrt{10}}*frac{10}{sqrt{10}}=-3;$$
Котангенс можно найти аналогично по формуле:
$$ctg(alpha)=frac{cos(alpha)}{sin(alpha)};$$
Но поступим проще и воспользуемся тригонометрической формулой, связывающей тангенс с котангенсом:
$$mathbf{сtg(alpha)=frac{1}{tg(alpha)};}$$
$$сtg(alpha)=frac{1}{-3}=-frac{1}{3};$$

Ответ: (tg(alpha)=-3;) (ctg(alpha)=-frac{1}{3}.)

Как видите, чтобы найти тангенс или котангенс через косинус или синус, необходимо воспользоваться сразу двумя тригонометрическими формулами. Это не очень удобно, поэтому очень полезны тригонометрические формулы, связывающие тангенс с косинусом или котангенс с синусом напрямую:
$$mathbf{tg(alpha)^2+1=frac{1}{cos(alpha)^2};}$$
$$mathbf{ctg(alpha)^2+1=frac{1}{sin(alpha)^2};}$$

Вывод связи тангенса с косинусом и котангенса с синусом

Полезно знать, как они выводятся. Вывод, на самом деле, элементарный, с использованием основного тригонометрического тождества и определения тангенса через синус и косинус:
$$tg(alpha)^2+1=frac{1}{cos(alpha)^2};$$
$$left(frac{sin(alpha)}{cos(alpha)}right)^2+1=frac{1}{cos(alpha)^2};$$
Приводим левую часть к общему знаменателю:
$$frac{sin(alpha)^2}{cos(alpha)^2}+frac{cos(alpha)^2}{cos(alpha)^2}=frac{1}{cos(alpha)^2};$$
$$frac{sin(alpha)^2+cos(alpha)^2}{cos(alpha)^2}=frac{1}{cos(alpha)^2};$$
В числителе у нас получилось основное тригонометрическое тождество:
$$frac{1}{cos(alpha)^2}=frac{1}{cos(alpha)^2};$$
Получилось верное равенство – формула доказана. Аналогично доказывается формула для котангенса и синуса. (В качестве упражнения докажите ее сами).

Если решать пример №2 по этим формулам, то решение заметно сокращается:
$$tg(alpha)^2+1=frac{1}{left(frac{sqrt{10}}{10}right)^2};$$
$$tg(alpha)^2+1=10;$$
$$tg(alpha)^2=9;$$
$$tg(alpha)=pm3;$$
Так как (alpha in (frac{3pi}{2};2pi)), то тангенс будет отрицательным:
$$tg(alpha)=-3;$$

Формулы суммы и разности тригонометрических функций

  1. Синус суммы и разности:
    $$mathbf{sin(alpha+beta)=sin(alpha)*cos(beta)+sin(beta)*cos(alpha);}$$
    $$mathbf{sin(alpha-beta)=sin(alpha)*cos(beta)-sin(beta)*cos(alpha);}$$
  2. Косинус суммы и разности:
    $$mathbf{cos(alpha+beta)=cos(alpha)*cos(beta)-sin(beta)*sin(alpha);}$$
    $$mathbf{cos(alpha-beta)=cos(alpha)*cos(beta)+sin(beta)*sin(alpha);}$$
  3. Тангенс суммы и разности:
    $$mathbf{tg(alpha+beta)=frac{tg(alpha)+tg(beta)}{1-tg(alpha)*tg(beta)};}$$
    $$mathbf{tg(alpha-beta)=frac{tg(alpha)-tg(beta)}{1+tg(alpha)*tg(beta)};}$$
  4. Котангенс суммы и разности:
    $$mathbf{сtg(alpha+beta)=frac{-1+сtg(alpha)*ctg(beta)}{ctg(alpha)+ctg(beta)};}$$
    $$mathbf{сtg(alpha-beta)=frac{-1-сtg(alpha)*ctg(beta)}{ctg(alpha)-ctg(beta)};}$$

Формулы суммы разности тригонометрических функций попадаются в ЕГЭ по профильной математике в №12. В прошлые года эти формулы давались в справочные материалах и учить их было не обязательно. Тем не менее, я бы рекомендовал выучить хотя бы формулы суммы и разности для синуса и косинуса.

Это не очень удобно, но иногда формулы суммы разности используют для вывода формул приведения:

Пример 3
Упростить выражение (sin(frac{pi}{2}+alpha)).

Воспользуемся формулой синуса суммы:
$$sin(alpha+beta)=sin(alpha)*cos(beta)+sin(beta)*cos(alpha);$$
$$sin(frac{pi}{2}+alpha)=sin(frac{pi}{2})*cos(alpha)+sin(alpha)*cos(frac{pi}{2})=$$
$$=1*cos(alpha)+sin(alpha)*0=cos(alpha);$$

Формулы суммы разности так же полезны, когда нужно посчитать значение тригонометрических функций некоторых нестандартных углов:

Пример 4
Найдите значение (sin(15^o)=?)

(15^o) нестандартный угол, вы его не найдете в тригонометрической таблице углов. Представим (15^o) в виде разности стандартных углов (15^o=45^o-30^o). И воспользуемся формулой синуса разности:
$$sin(alpha-beta)=sin(alpha)*cos(beta)-sin(beta)*cos(alpha);$$
$$sin(15^o)=sin(45^o-30^o)=sin(45^o)*cos(30^o)-sin(30^o)*cos(45^o)=$$
$$=frac{sqrt{2}}{2}*frac{sqrt{3}}{2}-frac{1}{2}*frac{sqrt{2}}{2}=$$
$$=frac{sqrt{6}}{4}-frac{sqrt{2}}{4}=frac{sqrt{6}-sqrt{2}}{4};$$
Вот мы наши синус (15^o). Получилось такое иррациональное некрасивое выражение, так и оставляем.

Ответ: (sin(15^o)=frac{sqrt{6}-sqrt{2}}{4}.)

Пример 5
Найдите значение (cos(75^o)=?)

(75^o) можно представить в виде суммы стандартных углов (75^o=30^o+45^o). Здесь воспользуемся формулой косинуса суммы:
$$cos(alpha+beta)=cos(30^o)*cos(45^o)-sin(30^0)*sin(45^0)=$$
$$=frac{sqrt{3}}{2}*frac{sqrt{2}}{2}-frac{1}{2}*frac{sqrt{2}}{2}=$$
$$=frac{sqrt{6}}{4}-frac{sqrt{2}}{4}=frac{sqrt{6}-sqrt{2}}{4};$$
У нас получился опять отвратительный ответ, но внимательный читатель заметит, что ответ такой же, как в предыдущем примере, это значит, что (cos(75^o)=sin(15^o)). Такой же вывод можно было бы сделать исходя из формул приведения и знания тригонометрической окружности.

Ответ: (cos(75^o)=frac{sqrt{6}-sqrt{2}}{4}.)

Мы не будем выводить эти формулы – это не самое приятное занятие. Их проще выучить, а вывод вам вряд ли когда-либо пригодится. Но сами формулы суммы и разности служат основой для доказательства других тригонометрических формул.

Формулы двойного угла

$$cos(2*alpha)=cos(alpha)^2-sin(alpha)^2=1-2*sin(alpha)^2=2*cos(alpha)^2-1;$$
$$sin(2*alpha)=2*sin(alpha)*cos(alpha);$$
$$tg(2*alpha)=frac{2*tg(alpha)}{1-tg(alpha)^2};$$
$$ctg(2*alpha)=frac{ctg(alpha)^2-1}{2*ctg(alpha)};$$

Формулы двойного угла для синуса, косинуса, тангенса и котангенса дают возможность выразить двойной угол (2alpha) через (alpha). Формулы для синуса и косинуса очень часто встречаются на ЕГЭ. Их обязательно нужно знать. Все они легко выводятся из формул синуса и косинуса суммы (формулы №5 и №6) :

$$cos(2alpha)=cos(alpha+alpha)=cos(alpha)*cos(alpha)-sin(alpha)*sin(alpha)=cos(alpha)^2-sin(alpha)^2;$$
Воспользовавшись основным тригонометрическим тождеством можно преобразовать эту формулу:
$$cos(2alpha)=cos(alpha)^2-sin(alpha)^2=1-sin(alpha)^2-sin(alpha)^2=1-2sin(alpha)^2;$$
$$cos(2alpha)=cos(alpha)^2-sin(alpha)^2=cos(alpha)^2-(1-cos(alpha)^2)=2cos(alpha)^2-1;$$

Синус двойного угла выводится аналогичным образом только с использованием формулы синуса суммы:
$$sin(2alpha)=sin(alpha)*cos(alpha)+sin(alpha)*cos(alpha)=2sin(alpha)cos(alpha);$$

Для вывода формул двойного угла для тангенса нам понадобится представить тангенс в виде отношения синуса к косинуса по определению и только что выведенные формулы синуса и косинуса двойного угла:
$$tg(2alpha)=frac{sin(2alpha)}{cos(2alpha)}=frac{2sin(alpha)cos(alpha)}{cos(alpha)^2-sin(alpha)^2}=frac{frac{2sin(alpha)cos(alpha)}{cos(alpha)^2}}{frac{cos(alpha)^2-sin(alpha)^2}{cos(alpha)^2}}=frac{frac{2sin(alpha)}{cos(alpha)}}{1-frac{sin(alpha)^2}{cos(alpha)^2}}=frac{2tg(alpha)}{1-tg(alpha)^2};$$
Котангенс двойного угла выводится абсолютно также:
$$сtg(2alpha)=frac{cos(2alpha)}{sin(2alpha)}=frac{cos(alpha)^2-sin(alpha)^2}{2sin(alpha)cos(alpha)}=frac{frac{cos(alpha)^2-sin(alpha)^2}{sin(alpha)^2}}{frac{2sin(alpha)cos(alpha)}{sin(alpha)^2}}=frac{frac{cos(alpha)^2}{sin(alpha)^2}-1}{frac{2cos(alpha)}{sin(alpha)}}=frac{ctg(alpha)^2-1}{2ctg(alpha)};$$

В первой части на ЕГЭ попадаются номера на преобразование тригонометрических выражений, где часто содержится двойной угол:

Пример 6
Найти значение (24cos(2alpha)=?), если (sin(alpha)=-0,2.)

Воспользуемся формулой косинуса двойного угла:
$$cos(2alpha)=1-2sin(alpha)^2;$$
$$24cos(2alpha)=24(1-2sin(alpha)^2)=24-48sin(alpha)^2=24-48*(-0,2)^2=24-48*0,04=22,08.$$

Пример 7
Найти значение (frac{10sin(6alpha)}{3cos(3alpha)}=?), если (sin(3alpha)=0,6.)

Используем синус двойного угла, для этого представим (6alpha=2*(3alpha)):
$$sin(6alpha)=sin(2*(3alpha))=2sin(3alpha)cos(3alpha);$$
$$frac{10sin(6alpha)}{3cos(3alpha)}=frac{10*2sin(3alpha)cos(3alpha)}{3cos(3alpha)}=frac{20sin(3alpha)}{3}=frac{20*0,6}{3}=frac{12}{3}=4.$$

Пример 8
Найти значение выражения (frac{12sin(11^o)cos(11^o)}{sin(22^o)}=?)

Замечаем, что (22^o=2*11^o) и воспользуемся синусом двойного угла:
$$frac{12sin(11^o)cos(11^o)}{sin(22^o)}=frac{12sin(11^o)cos(11^o)}{2sin(11^o)cos(11^o)}=frac{12}{2}=6.$$

Формулы тройного угла

Формулы тройного угла обычно попадаются на математических олимпиадах или вступительных экзаменах в математические ВУЗы. Учить их необязательно, но знать о существовании полезно, тем более, что они достаточно легко выводятся.
$$cos(3*alpha)=cos(alpha)^3-3*sin(alpha)^2*cos(alpha)=-3*cos(alpha)+4*cos(alpha)^3;$$
$$sin(3*alpha)=3*sin(alpha)*cos(alpha)^2-sin(alpha)^3=3*sin(alpha)-4*sin(alpha)^3;$$
$$tg(3*alpha)=frac{3*tg(alpha)-tg(alpha)^3}{1-3*tg(alpha)^2};$$
$$ctg(3*alpha)=frac{ctg(alpha)^3-3*ctg(alpha)}{3*ctg(alpha)^2-1};$$

Выведем эти формулы, использую формулы сложения. Начнем с косинуса тройного угла:
$$cos(3*alpha)=cos(2alpha+alpha)=cos(2alpha)*cos(alpha)-sin(2alpha)*sin(alpha)=$$
$$=(cos(alpha)^2-sin(alpha)^2)*cos(alpha)-2sin(alpha)*cos(alpha)*sin(alpha)=$$
$$=cos(alpha)^3-sin(alpha)^2*cos(alpha)-2sin(alpha)^2*cos(alpha)=$$
$$=cos(alpha)^3-3sin(alpha)^2*cos(alpha);$$

Если расписать (sin(alpha)^2=1-cos(alpha)^2), то получим еще один вариант формулы тройного угла:
$$cos(3*alpha)=cos(alpha)^3-3sin(alpha)^2*cos(alpha)=cos(alpha)^3-3(1-cos(alpha)^2)*cos(alpha)=$$
$$=4cos(alpha)^3-3cos(alpha);$$

Аналогично выводится формула синуса тройного угла:
$$sin(3alpha)=sin(2alpha+alpha)=sin(2alpha)*cos(alpha)+sin(alpha)*cos(2alpha)=$$
$$=2sin(alpha)*cos(alpha)*cos(alpha)+sin(alpha)*(cos(alpha)^2-sin(alpha)^2)=$$
$$=2sin(alpha)*cos(alpha)^2+sin(alpha)*cos(alpha)^2-sin(alpha)^3=3sin(alpha)*cos(alpha)^2-sin(alpha)^3;$$
Распишем по основному тригонометрическому тождеству (cos(alpha)^2=1-sin(alpha)^2) и подставим:
$$sin(3alpha)=3sin(alpha)*cos(alpha)^2-sin(alpha)^3=$$
$$=3sin(alpha)*(1-sin(alpha)^2)-sin(alpha)^3=3sin(alpha)-4sin(alpha)^3;$$

Для тангенса и котангенса формулы тройного угла здесь выводить не будем, так как они достаточно редки. Но в качестве упражнения можете сами выполнить вывод, представив тангенс или котангенс по определению: через отношение синуса тройного угла к косинусу тройного угла или наоборот соотвественно.

Формулы тройного угла обычно используются при преобразовании сложных тригонометрических выражений. Например, на вступительных экзаменах в МФТИ любят давать тригонометрические уравнения на тройной угол и больше.

Формулы половинного угла (двойного аргумента)

$$sin(frac{alpha}{2})^2=frac{1-cos(alpha)}{2};$$
$$cos(frac{alpha}{2})^2=frac{1+cos(alpha)}{2};$$
$$tg(frac{alpha}{2})^2=frac{1-cos(alpha)}{1+cos(alpha)};$$
$$ctg(frac{alpha}{2})^2=frac{1+cos(alpha)}{1-cos(alpha)};$$

Формулы половинного угла это по сути формулы обратные формулам двойного угла. Достаточно запомнить их элементарный вывод, тогда учить совсем необязательно. Здесь важный момент, что любой угол (alpha) всегда можно представить в виде удвоенного угла (frac{alpha}{2}):
$$alpha=2*frac{alpha}{2};$$

Выведем формулу синуса половинного угла, для этого нам понадобится формула косинуса двойного угла:
$$cos(alpha)=1-2*sin(frac{alpha}{2})^2;$$
Выразим отсюда (sin(frac{alpha}{2})):
$$sin(frac{alpha}{2})^2=frac{1-cos(alpha)}{2};$$
Иногда эту формулу записывают без квадрата:
$$sin(frac{alpha}{2})=pmsqrt{frac{1-cos(alpha)}{2}};$$
Плюс минус возникает при избавлении от квадрата.
Вывод косинуса половинного угла тоже получается из формулы косинуса двойного угла:
$$cos(alpha)=2*cos(frac{alpha}{2})^2-1;$$
$$cos(frac{alpha}{2})^2=frac{cos(alpha)+1}{2};$$
$$cos(frac{alpha}{2})=pmsqrt{frac{cos(alpha)+1}{2}};$$

Доказательство формул половинного угла для тангенса и котангенса следует из выше доказанных формул:
$$tg(frac{alpha}{2})=frac{sin(frac{alpha}{2})}{cos(frac{alpha}{2})}=frac{pmsqrt{frac{1-cos(alpha)}{2}}}{pmsqrt{frac{cos(alpha)+1}{2}}}=sqrt{frac{frac{1-cos(alpha)}{2}}{frac{cos(alpha)+1}{2}}}=frac{1-cos(alpha)}{1+cos(alpha)};$$
Точно так же для котангенса:
$$сtg(frac{alpha}{2})=frac{cos(frac{alpha}{2})}{sin(frac{alpha}{2})}=frac{pmsqrt{frac{cos(alpha)+1}{2}}}{pmsqrt{frac{1-cos(alpha)}{2}}}=sqrt{frac{frac{cos(alpha)+1}{2}}{frac{1-cos(alpha)}{2}}}=frac{1+cos(alpha)}{1-cos(alpha)};$$

Пример 9
При помощи формул половинного угла можно, например, посчитать (cos(15^o)):

$$cos(frac{alpha}{2})^2=frac{1+cos(alpha)}{2};$$
$$cos(15^o)^2=frac{1+cos(30^o)}{2}=frac{1+frac{sqrt{3}}{2}}{2}=frac{2+sqrt{3}}{4};$$
$$cos(15^o)=sqrt{frac{2+sqrt{3}}{4}}.$$

Кстати, формулы половинного угла справедливы не только в явном виде, когда аргумент правой части формулы (alpha), а левой (frac{alpha}{2}). Но и в неявном, достаточно, чтобы аргумент правой части был больше аргумента левой в два раза:
$$sin(5alpha)=pmsqrt{frac{1-cos(10alpha)}{2}};$$

Формулы понижения степени

$$sin(alpha)^2=frac{1-cos(2*alpha)}{2};$$
$$cos(alpha)^2=frac{1+cos(2*alpha)}{2};$$
$$sin(alpha)^3=frac{3*sin(alpha)-sin(3*alpha)}{4};$$
$$cos(alpha)^3=frac{3*cos(alpha)+cos(3*alpha)}{4};$$
$$sin(alpha)^4=frac{3-4*cos(2*alpha)+cos(4*alpha)}{8};$$
$$cos(alpha)^4=frac{3+4*cos(2*alpha)+cos(4*alpha)}{8};$$

Формулы понижения второй степени на самом деле дублируют формулы половинного угла.

Формулы понижения третей степени перестановкой слагаемых дублируют формулы тройного угла.

Преобразование суммы и разности тригонометрических функций:

$$sin(alpha)+sin(beta)=2*sinleft(frac{alpha+beta}{2}right)*cosleft(frac{alpha-beta}{2}right);$$
$$sin(alpha)-sin(beta)=2*sinleft(frac{alpha-beta}{2}right)*cosleft(frac{alpha+beta}{2}right);$$
$$cos(alpha)+cos(beta)=2*cosleft(frac{alpha+beta}{2}right)*cosleft(frac{alpha-beta}{2}right);$$
$$cos(alpha)-cos(beta)=-2*sinleft(frac{alpha+beta}{2}right)*sinleft(frac{alpha-beta}{2}right);$$
$$cos(alpha)-cos(beta)=2*sinleft(frac{alpha+beta}{2}right)*sinleft(frac{beta-alpha}{2}right);$$
$$tg(alpha)+tg(beta)=frac{sin(alpha+beta)}{cos(alpha)*cos(beta)};$$
$$tg(alpha)-tg(beta)=frac{sin(alpha-beta)}{cos(alpha)*cos(beta)};$$
$$ctg(alpha)+ctg(beta)=frac{sin(alpha+beta)}{sin(alpha)*sin(beta)};$$
$$ctg(alpha)-ctg(beta)=frac{sin(beta-alpha)}{sin(alpha)*sin(beta)};$$

Формулы для суммы и разности тригонометрических функций полезны, если необходимо превратить сумму двух функций в произведение. Они в основном используются в уравнениях и преобразованиях сложных выражений, когда необходимо слагаемые разложить на множители.

Для вывода формул суммы и разности синусов и косинусов нам понадобится пара трюков и формулы синуса и косинуса суммы и разности (тут можно запутаться, в названиях формул, будьте внимательны). Вывод получается не самый очевидный.

Обратите внимание, что любой угол (alpha) можно представить в таком странном виде:
$$alpha=frac{alpha}{2}+frac{alpha}{2}+frac{beta}{2}-frac{beta}{2}=frac{alpha+beta}{2}+frac{alpha-beta}{2};$$
Аналогично угол (beta):
$$beta=frac{alpha+beta}{2}-frac{alpha-beta}{2};$$
Эти странности нам понадобятся при выводе формул, просто обратите на них внимание.
А теперь перейдем непосредственно к выводу формулы суммы синусов двух углов. Для начала распишем угла (alpha) и (beta) по формулам выше:
$$sin(alpha)+sin(beta)=sin(frac{alpha+beta}{2}+frac{alpha-beta}{2})+sin(frac{alpha+beta}{2}-frac{alpha-beta}{2}); qquad (*)$$
Теперь воспользуемся формулами синуса суммы и синуса разности:

$$sin(gamma+sigma)=sin(gamma)*cos(sigma)+sin(sigma)*cos(gamma);$$
$$sin(gamma-sigma)=sin(gamma)*cos(sigma)-sin(sigma)*cos(gamma);$$

Только у нас под синусами будут стоять не (gamma) и (sigma), а целые выражения.
Пусть:
$$gamma=frac{alpha+beta}{2};$$
$$sigma=frac{alpha-beta}{2};$$
Применим формулы синуса суммы и разности в (*):
$$sin(alpha)+sin(beta)=sin(frac{alpha+beta}{2}+frac{alpha-beta}{2})+sin(frac{alpha+beta}{2}-frac{alpha-beta}{2})=$$
$$=left(sin(frac{alpha+beta}{2})*cos(frac{alpha-beta}{2})+sin(frac{alpha-beta}{2})*cos(frac{alpha+beta}{2})right)+$$
$$+left(sin(frac{alpha+beta}{2})*cos(frac{alpha-beta}{2})-sin(frac{alpha-beta}{2})*cos(frac{alpha+beta}{2})right)=$$
$$=2*sin(frac{alpha+beta}{2})*cos(frac{alpha-beta}{2}); $$
В самом конце мы просто раскрыли большие скобки и привели подобные слагаемые.

Аналогично выводятся все остальные формулы.

Пример 10
Вычислить (sin(165)+sin(75)=?)

(165^o) и (75^o) это не табличные углы. Значения синусов этих углов мы не знаем. Для решения этого примера воспользуемся формулой суммы синусов:
$$sin(alpha)+sin(beta)=2*sinleft(frac{alpha+beta}{2}right)*cosleft(frac{alpha-beta}{2}right);$$
$$sin(165^o)+sin(75^o)=2*sinleft(frac{165^o+75^o}{2}right)*cosleft(frac{165^o-75^o}{2}right)=$$
$$=2*sin(120^o)*cos(45^o)=2*frac{sqrt{3}}{2}*frac{sqrt{2}}{2}=frac{sqrt{6}}{2}.$$

Преобразование произведения тригонометрических функций

$$sin(alpha)*sin(beta)=frac{1}{2}*left(cos(alpha-beta)-cos(alpha+beta)right);$$
$$cos(alpha)*cos(beta)=frac{1}{2}*left(cos(alpha-beta)+cos(alpha+beta)right);$$
$$sin(alpha)*cos(beta)=frac{1}{2}*left(sin(alpha-beta)+sin(alpha+beta)right);$$

В некотором смысле формулы произведения синуса, косинуса, тангенса и котангенса являются обратными к тригонометрическим формулам суммы и разности тригонометрических функций. При помощи этих формул возможно перейти от произведения к сумме или разности.

Для вывода нам опять понадобятся формулы косинуса суммы и разности:
$$cos(alpha+beta)=cos(alpha)*cos(beta)-sin(beta)*sin(alpha);$$
$$cos(alpha-beta)=cos(alpha)*cos(beta)+sin(beta)*sin(alpha);$$

Сложим эти две формулы. Для этого складываем их левые части и приравниваем сумме правых частей:

$$cos(alpha+beta)+cos(alpha-beta)=cos(alpha)*cos(beta)-sin(beta)*sin(alpha)+cos(alpha)*cos(beta)+sin(beta)*sin(alpha);$$
Приводим подобные слагаемые:
$$cos(alpha+beta)+cos(alpha-beta)=2*cos(alpha)*cos(beta);$$
Отсюда получаем:
$$cos(alpha)*cos(beta)=frac{1}{2}*(cos(alpha+beta)+cos(alpha-beta));$$
Формула произведения косинусов доказана.

Произведение синусов доказывается похожим образом. Для этого домножим формулу косинуса суммы слева и справа на ((-1)):
$$-cos(alpha+beta)=-cos(alpha)*cos(beta)+sin(beta)*sin(alpha);$$
Косинус разности оставим без изменений:
$$cos(alpha-beta)=cos(alpha)*cos(beta)+sin(beta)*sin(alpha);$$
Сложим опять эти две формулы:
$$cos(alpha-beta)-cos(alpha+beta)=cos(alpha)*cos(beta)+sin(beta)*sin(alpha)-cos(alpha)*cos(beta)+sin(beta)*sin(alpha);$$
$$cos(alpha-beta)-cos(alpha+beta)=2*sin(beta)*sin(alpha);$$
$$sin(beta)*sin(alpha)=frac{1}{2}*(cos(alpha-beta)-cos(alpha+beta));$$
Произведение синусов тоже доказано.

Для того, чтобы вывести формулу произведения синуса и косинуса, нам понадобятся формулы синуса суммы и разности:
$$sin(alpha+beta)=sin(alpha)*cos(beta)+sin(beta)*cos(alpha);$$
$$sin(alpha-beta)=sin(alpha)*cos(beta)-sin(beta)*cos(alpha);$$
Сложим их:
$$sin(alpha+beta)+sin(alpha-beta)=sin(alpha)*cos(beta)+sin(beta)*cos(alpha)+sin(alpha)*cos(beta)-sin(beta)*cos(alpha);$$
$$sin(alpha+beta)+sin(alpha-beta)=2*sin(alpha)*cos(beta);$$
$$sin(alpha)*cos(beta)=frac{1}{2}*(sin(alpha+beta)+sin(alpha-beta));$$

Пример 11
Вычислить (sin(75^o)*cos(15^o)=?)

Воспользуемся формулой произведения синуса и косинуса:
$$sin(alpha)*cos(beta)=frac{1}{2}*(sin(alpha+beta)+sin(alpha-beta));$$
$$sin(75^o)*cos(15^o)=frac{1}{2}*(sin(75^o+15^o)+sin(75^o-15^o))=$$
$$=frac{1}{2}*(sin(90^o)+sin(60^o))=frac{1}{2}*(1+frac{sqrt{3}}{2})=frac{2+sqrt{3}}{4}.$$

Тригонометрия – раздел математической науки, в котором изучаются тригонометрические функции и их использование в геометрии. Развитие тригонометрии началось еще во времена античной Греции. Во времена средневековья важный вклад в развитие этой нужной науки внесли ученые Ближнего Востока и Индии, которые придумали наиболее важные понятия, объяснили многие свойства, предложили варианты измерения и др.

Данная статья посвящена базовым понятиям и дефинициям тригонометрии. В ней рассмотрены определения основных тригонометрических функций: синуса, косинуса, тангенса и котангенса. Разъяснен и проиллюстрирован их смысл в контексте геометрии без таблиц и графиков.

Синус, косинус, тангенс и котангенс. Определения

Зачем разделять понятия синуса, косинуса, тангенса и котангенса?

Изначально определения тригонометрических функций, аргументом которых является угол, выражались через соотношения сторон прямоугольного треугольника.

Определения тригонометрических функций

Что такое синус?

Синус угла (sin α) – это отношение противолежащего этому углу катета к гипотенузе.

Что такое косинус?

Косинус угла (cosα) – это отношение прилежащего катета к гипотенузе.

Что такое тангенс?

Тангенс угла (tg α) – это отношение противолежащего катета к прилежащему.

Котангенс угла (ctg α) – отношение прилежащего катета к противолежащему.

Данные определения даны для острого угла прямоугольного треугольника!

Синус и косинус можно представить через экспоненту (экспоненциальная функция).

Приведем иллюстрацию. 

Синус, косинус, тангенс и котангенс. Определения

В треугольнике ABC с прямым углом С синус угла А равен отношению катета BC к гипотенузе AB.

Означения синуса, косинуса, тангенса и котангенса позволяют вычислять (находить) значения этих функций по известным длинам сторон треугольника.

Что и почему важно и принято помнить в ходе такого нахождения?

Важно помнить!

Область значений синуса и косинуса: от -1 до 1. Иными словами синус и косинус принимают значения от -1 до 1. Область значений тг и ктг – вся числовая прямая, то есть эти функции могут принимать любые значения.

Как найти синус? Для начала нужно определиться, какой перед нами треугольник: прямоугольный или произвольный. В первом случае можно использовать обычный тригонометрический метод, а во втором – теорему косинусов.

Как найти косинус? Соответственно, нам нужно знать значения прилежающего катета и гипотенузы. 

Как найти тангенс? Если треугольник прямоугольный, то тангенс вычисляется при помощи значений противоположного катета и прилежащего (в уравнении нужно поделить одно на другое). Если речь идет о числах, тупых, развернутых углов и углов, превышающих 360 градусов, то тангенс определяется при помощи синуса и косинуса (посредством их отношения и деления).

Теорема синусов и косинусов используется для того чтобы искать элементы в произвольном треугольнике. Такой поиск используется часто.

Угол поворота

Определения, данные выше, относятся к острым углам. В тригонометрии вводится понятие угла поворота, величина которого, в отличие от острого угла, не ограничена рамками от 0 до 90 градусов.Угол поворота в градусах или радианах выражается любым действительным числом от -∞ до +∞. 

В данном контексте можно дать определение синуса, косинуса, тангенса и котангенса угла произвольной величины. Представим единичную окружность (круг) с центром в начале декартовой системы координат.

                                                                 Угол поворота

Начальная точка A с координатами (1, 0) поворачивается вокруг центра единичной окружности на некоторый угол α и переходит в точку A1. Определение дается через координаты точки A1(x , y). 

Синус (sin или син) угла поворота

Синус угла поворота α – это ордината точки A1(x , y). sin α=y

Косинус (cos) угла поворота

Косинус угла поворота α – это абсцисса точки A1(x , y). cos α=икс

Тангенс (tg) угла поворота

Тангенс угла поворота α – это отношение ординаты точки A1(x , y) к ее абсциссе. tg α=yx

Котангенс (ctg) угла поворота

Котанг угла поворота α – это отношение абсциссы точки A1(x , y) к ее ординате. ctg α=xy

Синус и косинус определены для любого угла поворота. Это логично, ведь абсциссу и ординату точки после поворота можно определить при любом угле. Иначе обстоит дело с тангенсом и котангенсом. Тангенс не определен, когда точка после поворота переходит в точку с нулевой абсциссой (0, 1) и (0, -1). В таких случаях выражение для тангенса tg α=yx просто не имеет смысла, так как в нем присутствует деление на ноль. Аналогична ситуация с котангенсом. Отличие состоит в том, что котангенс не определен в тех случаях, когда в ноль обращается ордината точки.

Важно помнить!

Простое правило: синус и косинус определены для любых углов α.

Тангенс определен для всех углов, кроме α=90°+180°·k, k∈Z (α=π2+π·k, k∈Z)

Котангенс определен для всех углов, кроме α=180°·k, k∈Z (α=π·k, k∈Z)

При решении практических примеров не говорят “синус угла поворота α”. Слова “угол поворота” просто опускают, подразумевая, что из контекста и так понятно, о чем идет речь. 

Числа

Как быть с определением синуса, косинуса, тангенса и котангенса числа, а не угла поворота?

Синус, косинус, тангенс, котангенс числа

Синусом, косинусом, тангенсом и котангенсом числа t называется число, которое соответственно равно синусу, косинусу, тангенсу и котангенсу в радиан.

Например, синус числа 10π равен синусу угла поворота величиной 10π рад.

Существует и другой подход к определению синуса, косинуса, тангенса и котангенса числа. Рассмотрим его подробнее.

Любому действительному числу t ставится в соответствие точка на единичной окружности с центром в начале прямоугольной декартовой системы координат. Синус, косинус, тангенс и котангенс определяются через координаты этой точки.

Начальная точка на окружности – точка A c координатами (1, 0).

Положительному числу t соответствует точка, в которую перейдет начальная точка, если будет двигаться по окружности против часовой стрелки и пройдет путь t.

Отрицательному числу t соответствует точка, в которую перейдет начальная точка, если будет двигаться по окружности против часовой стрелки и пройдет путь t.

Теперь, когда связь числа и точки на окружности установлена, переходим к определению синуса, косинуса, тангенса и котангенса.

Синус (sin) числа t

Синус числа t – ордината точки единичной окружности, соответствующей числу t. sin t=y

Косинус (cos) числа t

Косинус числа t – абсцисса точки единичной окружности, соответствующей числу t. cos t=x

Тангенс (tg) числа t

Тангенс числа t – отношение ординаты к абсциссе точки единичной окружности, соответствующей числу t. tg t=yx=sin tcos t

Последние определения находятся в соответствии и не противоречат определению, данному в начале это пункта. Точка на окружности, соответствующая числу t, совпадает с точкой, в которую переходит начальная точка после поворота на угол радиан.

Тригонометрические функции углового и числового аргумента

Каждому значению угла α соответствует определенное значение синуса и косинуса этого угла. Также, как всем углам α, отличным от α = 90 ° + 180 ° · k ,   k ∈ Z   ( α = π 2 + π · k ,   k ∈ Z ) соответствует определенное значение тангенса. Котангенс, как сказано выше, определен для всех α, кроме α = 180 ° · k ,   k ∈ Z   ( α = π · k ,   k ∈ Z ). 

Можно сказать, что sin α, cos α, tg α, ctg α – это функции угла альфа, или функции углового аргумента. 

Аналогично можно говорить о синусе, косинусе, тангенсе и котангенсе, как о функциях числового аргумента. Каждому действительному числу соответствует определенное значение синуса или косинуса числа t. Всем числам, отличным от π 2 + π · k ,   k ∈ Z соответствует значение тангенса. Котангенс, аналогично, определен для всех чисел, кроме π · k ,   k ∈ Z.

Основные функции тригонометрии

Синус, косинус, тангенс и котангенс – основные тригонометрические функции.

Из контекста обычно понятно, с каким аргументом тригонометрической функции (угловой аргумент или числовой аргумент) мы имеем дело. 

Связь определений sin, cos, tg и ctg из геометрии и тригонометрии

Вернемся к данным в самом начале определениям и углу альфа, лежащему в пределах от 0 до 90 градусов. Тригонометрические определения синуса, косинуса, тангенса и котангенса полностью согласуются с геометрическими определениями, данными с помощью  соотношений сторон прямоугольного треугольника. Покажем это.

                                                                     Связь определений sin, cos, tg и ctg из геометрии и тригонометрии

Возьмем единичную окружность с центром в прямоугольной декартовой системе координат. Повернем начальную точку A(1,0) на угол величиной до 90 градусов и проведем из полученной точки A1(x,y) перпендикуляр к оси абсцисс. В полученном прямоугольном треугольнике угол A1OH равен углу поворота α, длина катета OH равна абсциссе точки A1(x,y). Длина катета, противолежащего углу, равна ординате точки A1(x,y), а длина гипотенузы равна единице, так как она является радиусом единичной окружности. 

В соответствии с определением из геометрии, синус угла α равен отношению противолежащего катета к гипотенузе. 

sin α=A1HOA1=y1=y

Значит, определение синуса острого угла в прямоугольном треугольнике через соотношение сторон эквивалентно определению синуса угла поворота α, при альфа лежащем в пределах от 0 до 90 градусов.

Аналогично соответствие определений можно показать для косинуса, тангенса и котангенса.

Синус, косинус, тангенс и котангенс: основные формулы

Синус, косинус, тангенс и котангенс: основные формулы​​​​​​​

Для решения некоторых задач будет полезной таблица тригонометрических тождеств, которая позволит гораздо проще совершать преобразования функций:

Простейшие тригонометрические тождества

Простейшие тригонометрические тождества и определения тригонометрических функций.Преобразования sin cos tg

Частное от деления синуса угла альфа на косинус того же угла равно тангенсу этого угла (Формула 1). См. также доказательство правильности преобразования простейших тригонометрических тождеств. 
Частное от деления косинуса угла альфа на синус того же угла равно котангенсу этого же угла (Формула 2)
Секанс угла равен единице, деленной на косинус этого же самого угла (Формула 3)
Сумма квадратов синуса и косинуса одного и того же угла равна единице (Формула 4). см. также доказательство суммы квадратов косинуса и синуса.
Сумма единицы и тангенса угла равна отношению единицы к квадрату косинуса этого угла (Формула 5)
Единица плюс котангенс угла равна частному от деления единицы на синус квадрат этого угла (Формула 6)
Произведение тангенса на котангенс одного и того же угла равно единице (Формула 7).

Преобразование отрицательных углов тригонометрических функций (четность и нечетность)

Для того, чтобы избавиться от отрицательного значения градусной меры угла при вычислении синуса, косинуса или тангенса, можно воспользоваться следующими тригонометрическими преобразованиями (тождествами), основанными на принципах четности или нечетности тригонометрических функций.

Формулы преобразования отрицательных значений углов аргумента тригонометрических функций.Преобразование синуса минус альфа, косинуса, тангенса и котангенса отрицательных углов

Как видно, косинус и секанс является четной функцией, синус, тангенс и котангенс – нечетные функции.

Синус отрицательного угла равен отрицательному значению синуса этого же самого положительного угла (минус синус альфа).
Косинус “минус альфа” даст тоже самое значение, что и косинус угла альфа.
Тангенс минус альфа равен минус тангенс альфа.

Формулы приведения двойного угла (синус, косинус, тангенс и котангенс двойного угла)

Если необходимо разделить угол пополам, или наоборот, перейти от двойного угла к одинарному, можно воспользоваться следующими тригонометрическими тождествами:

Формулы тригонометрических тождеств преобразования двойного угла альфа в одинарный. 2α в α

Преобразование двойного угла (синуса двойного угла, косинуса двойного угла и тангенса двойного угла) в одинарный происходит по следующим правилам:

Синус двойного угла равен удвоенному произведению синуса на косинус одинарного угла

Косинус двойного угла равен разности квадрата косинуса одинарного угла и квадрата синуса этого угла

Косинус двойного угла равен удвоенному квадрату косинуса одинарного угла минус единица

Косинус двойного угла равен единице минус двойной синус квадрат одинарного угла

Тангенс двойного угла равен дроби, числитель которой – удвоенный тангенс одинарного угла, а знаменатель равен единице минус тангенс квадрат одинарного угла.

Котангенс двойного угла равен дроби, числитель которой – квадрат котангенса одинарного угла минус единица, а знаменатель равен удвоенному котангенсу одинарного угла

Формулы универсальной тригонометрической подстановки

Указанные ниже формулы преобразования могут пригодиться, когда нужно аргумент тригонометрической функции ( sin α, cos α, tg α) разделить на два и привести выражение к значению половины угла. Из значения α получаем  α/2 .

Данные формулы называются формулами универсальной тригонометрической подстановки. Их ценность заключается в том, что тригонометрическое выражение с их помощью сводится к выражению тангенса половины угла, вне зависимости от того, какие тригонометрические функции (sin cos tg ctg) были в выражении изначально. После этого уравнение с тангенсом половины угла решить гораздо проще.
Тригонометрические тождества для приведения величины угла к его половине. (преобразование угла к половинному аргументу через тангенс)

Тригонометрические тождества преобразования половины угла

Указанные ниже формулы тригонометрического преобразования половинной величины угла к его целому значению.
Значение аргумента тригонометрической функции α/2 приводится к значению аргумента тригонометрической функции α.
Тригонометрические формулы (тождества) преобразования половины угла к целому значению. Формулы половины аргумента тригонометрической функции

Тригонометрические формулы сложения углов

Тригонометрические формулы (тождества) сложения углов, которые являются аргументом тригонометрической функции

cos (α – β) = cos α · cos β + sin α · sin β

sin (α + β) = sin α · cos β + sin β · cos α 

sin (α – β) = sin α · cos β – sin β · cos α 
cos (α + β) = cos α · cos β – sin α · sin β 

Тангенс и котангенс суммы углов альфа и бета могут быть преобразованы по следующим правилам преобразования тригонометрических функций:

Тангенс суммы углов равен дроби, числитель которой – сумма тангенса первого и тангенса второго угла, а знаменатель – единица минус произведение тангенса первого угла на тангенс второго угла.

Тангенс разности углов равен дроби, числитель которой равен разности тангенса уменьшаемого угла и тангенса вычитаемого угла, а знаменатель – единице плюс произведение тангенсов этих углов.

Котангенс суммы углов равен дроби, числитель которой равен произведению котангенсов этих углов плюс единица, а знаменатель равен разности котангенса второго угла и котангенса первого угла.

Котангенс разности углов равен дроби, числитель которой – произведение котангенсов этих углов минус единица, а знаменатель равен сумме котангенсов этих углов.

Данные тригонометрические тождества удобно применять, когда нужно вычислить, например, тангенс 105 градусов (tg 105). Если его представить как tg (45 + 60), то можно воспользоваться приведенными тождественными преобразованиями тангенса суммы углов, после чего просто подставить табличные значения тангенса 45 и тангенса 60 градусов.

Формулы преобразования суммы или разности тригонометрических функций

Выражения, представляющие собой сумму вида sin α + sin β можно преобразовать с помощью следующих формул:
Формулы преобразования суммы или разности тригонометрических функций двух разных углов в их произведение

Формулы тройного угла – преобразование sin3α cos3α tg3α в sinα cosα tgα

Иногда необходимо преобразовать тройную величину угла так, чтобы аргументом тригонометрической функции вместо 3α стал угол α.
В этом случае можно воспользоваться формулами (тождествами) преобразования тройного угла:
Формулы преобразования тройного угла тригонометрических функций. (Тождества преобразования 3a в a) sin3a cos3a tg3a ctg3a

Формулы преобразования произведения тригонометрических функций

Если возникает необходимость преобразовать произведение синусов разных углов косинусов разных углов или даже произведения синуса на косинус, то можно воспользоваться следующими тригонометрическими тождествами:
Тригонометрические формулы преобразования произведения функций разных углов в их сумму или разность. Преобразование sin a * sin b, cos a * cos b, tg a * tg b
В этом случае произведение функций синуса, косинуса или тангенса разных углов будет преобразовано в сумму или разность.

Формулы приведения тригонометрических функций

Пользоваться таблицей приведения нужно следующим образом. В строке выбираем функцию, которая нас интересует. В столбце – угол. Например, синус угла (α+90) на пересечении первой строки и первого столбца выясняем, что sin (α+90)  = cos α .

См. также Полный список формул приведения тригонометрических функций.

Угол α + 90
α + π/2
α + 180
α + π
α + 270
α + 3π/2
90 – α
π/2- α
180 – α
π- α
270 – α
3π/2- α
360 – α
2π- α
sin cos α -sin α -cos α cos α sin α -cos α -sin α
cos -sin α -cos α sin α sin α -cos α -sin α cos α
tg -ctg α tg α -ctg α ctg α -tg α ctg α -tg α
ctg -tg α ctg α -tg α tg α -ctg α tg α -ctg α


0
 

 Начать курс обучения

Формулы тригонометрии

В этой статье мы изучим все тригонометрические формулы, которые могут понадобится на ЕГЭ.

От основного тригонометрического тождества, до формул тройного угла.

Мы решим вместе 22 примера, чтобы «набить руку» и уметь решать любые задачи.

Поехали!

Формулы тригонометрии — коротко о главном

Основные формулы:

Название формулы Формула
Основное тригонометрическое тождество (ночью разбудят — должен вспомнить!) ( displaystyle si{{n}^{2}}a+co{{s}^{2}}a=1)
Выражение тангенса через синус и косинус (по сути альтернативное определение тангенса) ( displaystyle tg alpha =frac{sin alpha }{cos alpha })
Выражение котангенса через синус и косинус или через тангенс (по сути альтернативное определение котангенса) ( displaystyle ctg alpha =frac{cos alpha }{sin alpha }=frac{1}{tg alpha })
Синус суммы и разности: ( displaystyle sin left( alpha pm beta right)=sinalpha cdot cosbeta pm cosalpha cdot sinbeta )
Косинус суммы и разности: ( displaystyle cos left( alpha pm beta right)=cosalpha cdot cosbeta mp sinalpha cdot sinbeta )
Тангенс суммы и разности: ( displaystyle tgleft( alpha pm beta right)=frac{tgalpha pm tgbeta }{1mp tgalpha cdot tgbeta })

Формулы понижения степени:

Данная группа формул позволяет перейти от любого тригонометрического выражения к рациональному.

  • ( displaystyle si{{n}^{2}}alpha =frac{1-cos2alpha }{2})
  • ( displaystyle co{{s}^{2}}alpha =frac{1+cos2alpha }{2})
  • ( displaystyle si{{n}^{3}}alpha =frac{3sinalpha -sin3alpha }{4})
  • ( displaystyle co{{s}^{3}}a=frac{3cosa+cos3a}{4})
  • ( displaystyle t{{g}^{2}}alpha =frac{1-cos2alpha }{1+cos2alpha },alpha ne frac{pi }{2}+pi n,nin Z)

Формулы преобразования функций:

Данная группа формул позволяет преобразовать произведение в сумму и сумму в произведение.

  • ( displaystyle sinalpha pm sinbeta =2sinfrac{alpha pm beta }{2}cosfrac{alpha mp beta }{2})
  • ( displaystyle cosalpha +cosbeta =2cosfrac{alpha +beta }{2}cosfrac{alpha -beta }{2})
  • ( displaystyle cosalpha -cosbeta =-2sinfrac{alpha +beta }{2}sinfrac{alpha -beta }{2})
  • ( displaystyle tgalpha pm tgbeta =frac{text{sin}left( alpha pm beta right)}{cosalpha cosbeta })
  • ( displaystyle ctgalpha pm ctgbeta =frac{text{sin}left( beta pm alpha right)}{sinalpha sinbeta })

Формулы преобразования произведений функций:

  • ( displaystyle sinalpha sinbeta =frac{cos left( alpha -beta right)-text{cos}left( alpha +beta right)}{2})
  • ( displaystyle sinalpha cosbeta =frac{sin left( alpha +beta right)+text{sin}left( alpha -beta right)}{2})
  • ( displaystyle cosalpha cosbeta =frac{cos left( alpha -beta right)+text{cos}left( alpha +beta right)}{2})

Таблица значений тригонометрических функций:

Тригонометрические функции

Как ты уже понял, тригонометрические выражения – это выражения, в котором переменная содержится под знаком тригонометрических функций. 

Стоп! Вот прямо здесь мы и остановимся! Я задам тебе вопрос: какие тригонометрические функции ты знаешь?

Верно! Их всего четыре!

  • Синус ( displaystyle sinleft( x right))
  • Косинус ( displaystyle cosleft( x right))
  • Тангенс ( displaystyle tgleft( x right))
  • Котангенс ( displaystyle ctgleft( x right))

Хотя, положа руку на сердце, я скажу тебе, что знание последней не так уж и обязательно (хотя желательно!), поскольку она легко выражается через тангенс.

Да и сам тангенс, по сути – тоже лишь тригонометрическое выражение, зависящее от синуса и косинуса.

Таким образом, у нас есть две основные тригонометрические функции – синус и косинус и две «второстепенные» – тангенс и котангенс.

Я не буду сейчас определять, что такое синус и косинус, ты и так это уже знаешь из предыдущих разделов. Я лишь скажу пару слов про важность этих понятий.

Итак, пара слов: первые зачатки тригонометрии возникли более 3 тысяч лет назад. Я думаю, что тебе очевидно, что тогда люди не занимались «формулами ради формул».

Так что тригонометрические функции имеют полезные практические свойства. Я не буду их перечислять. Если тебе интересно, ты всегда можешь найти море информации в интернете.

Если все, что я сказал выше, звучало для тебя древним эльфийским языком, то посмотри статью о тригонометрической окружности.

А сейчас я приведу тебе некоторые основные соотношения между тригонометрическими величинами, которые оказываются полезными при решении задач.

Таблица значений тригонометрических функций

Тебе нужно помнить таблицу значений тригонометрических функций для углов хотя бы первой четверти! Я сейчас нарисую здесь эту таблицу, а потом объясню тебе, как сделать ее запоминание проще.

Или ее расширенный вариант для всех «основных углов»:

Я ни в коей мере не настаиваю (и даже не надеюсь), что ты выучишь вторую таблицу. Сказать по правде, я и сам ее не знаю.

Но первую таблицу знать совершенно необходимо.

Не всегда на экзамене у тебя будет время, чтобы вывести самостоятельно, скажем, синус ( displaystyle 60) градусов.

Для того, чтобы запомнить первую таблицу можно поступить так:

Запомнить всего 5 значений для, скажем, синуса. Затем тебе не составит труда заметить, что для косинуса все значения идут «наоборот»:

  • Например, синус ( displaystyle 0) градусов равен нулю значит, косинус ( displaystyle 0) градусов – наоборот: единица.
  • Синус ( displaystyle 90) градусов равен единице, значит косинус ( displaystyle 90) градусов равен нулю.
  • Синус ( displaystyle 30) градусов равен ( displaystyle frac{1}{2}), значит косинус ( displaystyle 30) градусов равен ( displaystyle frac{sqrt{3}}{2}) и т. д.

Тангенс можно получить, разделив синус угла на косинус. Как же всегда вывести большую таблицу, зная малую, я тебе непременно расскажу чуть позднее.

Формулы тригонометрии (основа)

Название формулы Формула
Основное тригонометрическое тождество (ночью разбудят — должен вспомнить!) ( displaystyle si{{n}^{2}}a+co{{s}^{2}}a=1)
Выражение тангенса через синус и косинус (по сути альтернативное определение тангенса) ( displaystyle tg alpha =frac{sin alpha }{cos alpha })
Выражение котангенса через синус и косинус или через тангенс (по сути альтернативное определение котангенса) ( displaystyle ctg alpha =frac{cos alpha }{sin alpha }=frac{1}{tg alpha })
Первое следствие формулы 1: ( displaystyle t{{g}^{2}}alpha +1=frac{1}{co{{s}^{2}}alpha })
Второе следствие формулы 1: ( displaystyle ct{{g}^{2}}alpha +1=frac{1}{si{{n}^{2}}alpha })
Третье следствие формулы 1: ( displaystyle sin alpha =pm sqrt{1-co{{s}^{2}}alpha })
Четвертое следствие формулы 1: ( displaystyle cos alpha =pm sqrt{1-si{{n}^{2}}alpha })

Уже получилось 7 формул! К сожалению, это еще далеко не предел. Совсем не предел.

Тем не менее последние 4 формулы есть ни что иное, как простое следствие первой. В самом деле, ты заметил, почему это так?

Формула 4 получается делением обеих частей формулы 1 на ( displaystyle co{{s}^{2}}alpha ) и применением формулы 2.

Формула 5 получается аналогично: разделим обе части формулы 1 на ( displaystyle si{{n}^{2}}alpha ) и вместо выражения ( displaystyle frac{co{{s}^{2}}alpha }{si{{n}^{2}}alpha }) запишем ( displaystyle ct{{g}^{2}}alpha ), исходя из определения 3.

Формулы 1 – 5 мы трактуем вполне однозначно. Чего нельзя сказать про формулы 6 и 7. В чем «фишка» формул 6 и 7?

Их особенность заключается в знаке ( displaystyle pm ), который стоит перед корнем.

Как это понимать? А понимать надо так: в некоторых случаях мы ставим плюс, а в некоторых – минус.

Теперь у тебя должен возникнуть вопрос: в каких-таких «некоторых случаях»? Туманность этой формулировки снимается следующим правилом:

Если в формуле
( displaystyle sin alpha =pm sqrt{1-co{{s}^{2}}alpha })
угол ( displaystyle alpha ) таков, что ( displaystyle text{sin} text{ }!!alpha!!text{ }<0), то ставим знак «минус», иначе – «плюс».

Если в формуле
( displaystyle cos alpha =pm sqrt{1-si{{n}^{2}}alpha })
угол ( displaystyle alpha ) таков, что ( displaystyle text{cos} text{ }!!alpha!!text{ }<0), то ставим знак «минус», иначе – «плюс».

Есть опять некий «запутанный» момент в правиле, не так ли? В чем осталось разобраться?

Осталось понять, как связан угол со знаком тригонометрической функции. Ответом на этот вопрос (если ты, конечно, забыл) служат следующие картинки:

Они подскажут тебе, какой нужно выбирать знак для той или иной функции, так что ты не допустишь досадной ошибки.

К тому же это избавит тебя от мучительных размышлений по поводу того «а зачем в этом примере нужен этот угол?!».

4 примера на тренировку

  • Най­ди­те ( displaystyle text{3cos} text{ }!!alpha!!text{ }), если ( displaystyle sinalpha =-frac{2sqrt{2}}{3}) и ( displaystyle alpha in left( frac{3pi }{2};2pi right)).
  • Най­ди­те ( displaystyle 5sinalpha), если ( displaystyle cosalpha =frac{2sqrt{6}}{5}) и ( displaystyle alpha in left( frac{3pi }{2};2pi right)).
  • Най­ди­те ( displaystyle text{cos} text{ }!!alpha!!text{ }) если ( displaystyle sinalpha =frac{2sqrt{6}}{5}) и ( displaystyle alpha in left( frac{pi }{2};pi right)).
  •  Най­ди­те ( displaystyle text{tg} text{ }!!alpha!!text{ }), если ( displaystyle sinalpha =-frac{5}{sqrt{26}}) и ( displaystyle alpha in left( pi ;frac{3pi }{2} right)).

Решения:

1. Так как ( displaystyle cosalpha =pm sqrt{1-si{{n}^{2}}alpha }), то подставим сюда значение( displaystyle sinalpha =-frac{2sqrt{2}}{3}), тогда ( displaystyle cosalpha =pm sqrt{1-{{left( -frac{2sqrt{2}}{3} right)}^{2}}}=pm sqrt{1-frac{4cdot 2}{9}}=pm sqrt{1-frac{8}{9}}=)

( displaystyle=pm sqrt{frac{1}{9}}=pm frac{1}{3}.)

Теперь дело за малым: разобраться со знаком. Что нам для этого нужно? Знать, в какой четверти находится наш угол.

По условию задачи: ( displaystyle alpha in left( frac{3pi }{2};2pi right)). Смотри на картинку. Какая это четверть? Четвертая.

Каков знак косинуса в четвертой четверти? На картинке стоит знак «плюс», значит косинус в четвертой четверти положительный.

Тогда нам остается выбрать знак «плюс» перед ( displaystyle frac{1}{3}). ( displaystyle text{cos} text{ }!!alpha!!text{ }=frac{1}{3}), тогда ( displaystyle 3cosalpha =3cdot frac{1}{3}=1).

Ответ: ( displaystyle 1).

Ну вот видишь, ничего сложного. Абсолютно ничего. Нужно лишь запомнить знаки синуса, косинуса и тангенса (котангенса) по четвертям. Ну а как это делать автоматически описано в статье, посвященной тригонометрической окружности.

Давай разберем оставшиеся примеры.

2. Так как ( displaystyle sin alpha =pm sqrt{1-co{{s}^{2}}alpha }), то все, что нам нужно – это подставить ( displaystyle cosalpha =frac{2sqrt{6}}{5}) в нашу формулу. Что мы с тобой и сделаем:

( displaystyle sinalpha =pm sqrt{1-{{left( frac{2sqrt{6}}{5} right)}^{2}}}=pm sqrt{1-left( frac{4cdot 6}{25} right)}=pm sqrt{frac{1}{25}}=pm frac{1}{5}).

Опять нужно определиться со знаком. Смотрим на рисунок. Четверть – снова четвертая. Знак синуса четвертой четверти – отрицательный. Ставим знак «минус». ( displaystyle sinalpha =-frac{1}{5}), тогда ( displaystyle 5sinalpha =-5cdot frac{1}{5}=-1).

Ответ: ( displaystyle -1).

3. Ничего нового. Скорее для закрепления. Снова подставляем в формулу ( displaystyle cos alpha =pm sqrt{1-si{{n}^{2}}alpha }) значение ( displaystyle sinalpha =frac{2sqrt{6}}{5}):

( displaystyle cosalpha =pm sqrt{1-{{left( frac{2sqrt{6}}{5} right)}^{2}}}=pm sqrt{1-left( frac{4cdot 6}{25} right)}=pm sqrt{frac{1}{25}}=pm frac{1}{5}).

Смотрим на знак косинуса при ( displaystyle alpha in left( frac{pi }{2};pi right)). Какая это четверть? Вторая. Косинус второй четверти отрицательный. Тогда выбираем знак «минус».

Ответ: ( displaystyle -0,2).

4. Здесь перед нами стоит задачка чуть сложнее. Однако, не стоит огорчаться. Давай вспомним, что такое тангенс. Это ведь отношение синуса к косинусу. Синус нам уже дан.

Давай вначале найдем косинус. Как это сделать, ты уже знаешь. ( displaystyle cosalpha =pm sqrt{1-{{left( -frac{5}{sqrt{26}} right)}^{2}}}=pm sqrt{1-frac{25}{26}}=pm sqrt{frac{1}{26}}=pm frac{1}{sqrt{26}}).

Так как ( displaystyle alpha in left( pi ;frac{3pi }{2} right)) (это угол в третьей четверти, а косинус в третьей четверти имеет знак «минус»), то ( displaystyle cosalpha =-frac{1}{sqrt{26}}).

Теперь все, что нам осталось, это воспользоваться определением тангенса:

( displaystyle tgalpha =frac{sinalpha }{cosalpha }=frac{-frac{5}{sqrt{26}}}{-frac{1}{sqrt{26}}}=5.)

Ответ: ( displaystyle 5).

Уф, выдохнули! Ну вот мы с тобой решили некоторые (довольно типичные и распространенные) примеры. Ты спросишь: «И что, это все?». Я отвечу, что, увы нет. Это далеко не все.

Далее нам потребуются более сложные формулы тригонометрии.

Формулы тригонометрии (более сложные)

Название формулы Формула
Синус суммы и разности: ( displaystyle sin left( alpha pm beta right)=sinalpha cdot cosbeta pm cosalpha cdot sinbeta )
Косинус суммы и разности: ( displaystyle cos left( alpha pm beta right)=cosalpha cdot cosbeta mp sinalpha cdot sinbeta )
Тангенс суммы и разности: ( displaystyle tgleft( alpha pm beta right)=frac{tgalpha pm tgbeta }{1mp tgalpha cdot tgbeta })
Синус двойного угла (следствие формулы 1) ( displaystyle sin2a=2sinacdot cosa)
Косинус двойного угла (следствие формулы 2) ( displaystyle cos2a=co{{s}^{2}}a-si{{n}^{2}}a)
( displaystyle cos2a=2co{{s}^{2}}a-1=1-2si{{n}^{2}}a)
Тангенс двойного угла: ( displaystyle tg2a=frac{2tga}{1-t{{g}^{2}}a})

Как распознать, что тебе требуются именно эти, а не какие-нибудь другие формулы?

Очень просто: если ты видишь косинус, синус, тангенс от суммы двух углов или двойных углов, то это должно служить тебе индикатором – мне нужно применить одну из формул для суммы/разности или для двойного угла.

Звучит несколько путано? Давай посмотрим на примеры. Заодно я дам еще ряд важных комментариев.

9 примеров на тренировку

  • ( displaystyle frac{12sin11{}^circ cos11{}^circ }{sin22{}^circ })
  • ( displaystyle frac{24left( si{{n}^{2}}17{}^circ -co{{s}^{2}}17{}^circ right)}{cos34{}^circ })
  • ( displaystyle 36sqrt{6} ctg frac{pi}{6} sin frac{pi }{4})
  •  Най­ди­те ( displaystyle -47cos2a), если ( displaystyle cosa=-0,4)
  •  Най­ди­те ( displaystyle frac{10sin6a}{3cos3a}), если ( displaystyle sin3a=0,6)
  • Най­ди­те ( displaystyle 26text{cos}left( frac{3pi }{2}+a right)), если ( displaystyle cosa=frac{12}{13}) и ( displaystyle alpha in left( frac{3pi }{2};2pi right))
  •  Най­ди­те ( displaystyle t{{g}^{2}}a), если ( displaystyle 5si{{n}^{2}}a+13co{{s}^{2}}a=6)
  •  Най­ди­те ( displaystyle frac{10cosa+4sina+15}{2sina+5cosa+3}), если ( displaystyle tga=-2,5)
  • Най­ди­те ( displaystyle 7cos left( pi +beta right)-2text{sin}left( frac{pi }{2}+beta right)), если ( displaystyle cosbeta =-frac{1}{3})

Список этих заданий можно продолжать бесконечно… Но я выбрал здесь: а) не самые сложные формулы; б) не самые «страшные» углы.

Страшные углы я припас нам напоследок 🙂

Решения:

Кстати, здесь тебе понадобится знание также тех формул, которые я привел в самом начале. Поехали!

1. ( displaystyle frac{12sin11{}^circ cos11{}^circ }{sin22{}^circ })

Ни ты, ни я не знаем, чему в точности равен синус или косинус ( displaystyle 11) градусов, и чему равен синус ( displaystyle 22) градусов.

Но что мы должны заметить?

Верно! ( displaystyle 22{}^circ =2cdot 11{}^circ ). Значит, снизу записан синус двойного угла! Тогда применим формулу синуса двойного угла:

( displaystyle sin22{}^circ =2sin11{}^circ cdot cos11{}^circ )

Подставим это значение в знаменатель нашей дроби и сократим!

( displaystyle frac{12sin11{}^circ cdot cos11{}^circ }{sin22{}^circ }=frac{12sin11{}^circ cdot cos11{}^circ }{2sin11{}^circ cdot cos11{}^circ }=6).

Ответ: ( displaystyle 6).

Ну вот, ничего страшного не случилось? Пример решился в одну строчку с применением одной единственной формулы. Другое дело, иногда не совсем очевидно, какую из формул применять.

Тут тебе нужен опыт. Нужно, как говорится, «набить руку» на таких примерах.

2. ( displaystyle frac{24left( si{{n}^{2}}17{}^circ -co{{s}^{2}}17{}^circ right)}{cos34{}^circ })

Опять-таки, сразу можно заметить, что ( displaystyle 34{}^circ =2cdot 17{}^circ ). ( displaystyle 34) градуса стоит в косинусе. Это говорит о том, что в примере спрятан косинус двойного угла. Вспомним его определение:

( displaystyle cos2a=co{{s}^{2}}a-si{{n}^{2}}a)

Что же у нас есть в числителе? А там все наоборот: синус в квадрате вычитается из косинуса в квадрате. Тогда в числителе у нас написана формула чего?

3. ( displaystyle 36sqrt{6}ctgfrac{pi }{6}sinfrac{pi }{4})

Здесь нет ничего сложного, абсолютно ничего! Но есть одно «но!».

Это «но» заключается в том, что тебе нужно помнить таблицу значений тригонометрических функций для углов хотя бы первой четверти! (Как ее запомнить я рассказал ранее, а сейчас просто приведу ее еще раз).

Или ее расширенный вариант для всех «основных углов»:

И посмотрим в таблицу:

( displaystyle ctgfrac{pi }{6}=sqrt{3}), ( displaystyle sinfrac{pi }{4}=frac{sqrt{2}}{2}). Подставим эти значения в нашу формулу:

( displaystyle 36sqrt{6} ctgfrac{pi }{6}sinfrac{pi }{4}=36sqrt{6}cdot sqrt{3}cdot frac{sqrt{2}}{2}=frac{36cdot sqrt{6}cdot sqrt{6}}{2}=frac{36cdot 6}{2}=36cdot 3=108).

Ответ: ( displaystyle 108)

Вот видишь, знание первой таблицы совершенно необходимо! Без нее – вообще нет никакой тригонометрии. Так что, пожалуйста, будь добр, выучи.

Это не потребует от тебя значительных усилий и избавит от массы глупых ошибок в будущем. Еще раз специально скажу: большую таблицу учить не надо!!!

4. По условию (cosa=-0,4), нам же надо найти (-47cos2a).

Что тогда надо сделать?

Верно, наша цель – выразить косинус двойного угла через угол «одинарный». Есть ли такая формула? Конечно, есть! Вот она:

5. ( displaystyle frac{10sin6a}{3cos3a}) – это то, что надо вычислить, а ( displaystyle sin3a=0,6) – это то, что есть.

Ну что же, надо отталкиваться от того, что есть. Вроде бы этого должно быть достаточно. Здесь все опять несложно!

Нужно лишь заметить, что ( displaystyle sin6alpha =2sin3alpha cdot cos3alpha ). Давай это и подставим в числитель исходной дроби. Что же мы имеем?

6. ( displaystyle 26text{cos}left( frac{3pi }{2}+a right)) – то, что нужно найти, а ( displaystyle cosa=frac{12}{13}) и ( displaystyle alpha in left( frac{3pi }{2};2pi right)) – то, что мы имеем.

На самом деле здесь можно поступать двояко. Но о втором способе я скажу тебе чуть позже. А пока давай подумаем, что нужно найти.

А найти нужно по сути косинус от суммы двух углов. Причем один из них известен. Давай не будем долго думать и разложим косинус суммы на произведение:

( displaystyle cos left( frac{3pi }{2}+alpha right)=cosfrac{3pi }{2}cosalpha -sinfrac{3pi }{2}sinalpha )

Вспомни единичную окружность (ну или на худой конец посмотри в расширенную таблицу).

Косинус углов: ( displaystyle frac{pi }{2}=90{}^circ ,~frac{3pi }{2}=270{}^circ ) равен нулю!

Тогда…

7. Нужно найти: ( displaystyle t{{g}^{2}}a), а дано: ( displaystyle 5si{{n}^{2}}a+13co{{s}^{2}}a=6).

Тут все можно сделать только зная, что такое тангенс и основное тригонометрическое тождество. По порядку:

( displaystyle t{{g}^{2}}alpha =frac{si{{n}^{2}}alpha }{co{{s}^{2}}alpha }),
( displaystyle si{{n}^{2}}a+co{{s}^{2}}a=1)

Тогда решить задачу можно вот как: найти по отдельности значения синуса в квадрате и косинуса в квадрате, а затем при помощи полученных значений найти тангенс. Так мы с тобой и сделаем:

Вначале найдем синус в квадрате.

8. Надо найти ( displaystyle frac{10cosa+4sina+15}{2sina+5cosa+3}), зная, что ( displaystyle tga=-2,5).

На какую мысль тебя это должно было натолкнуть?

А на ту, что если нам дан тангенс, то и наше выражение нужно привести к такому виду, чтобы оно зависело от тангенсов, которые мы потом в него и подставим. Напомню тебе, что

( displaystyle tgalpha =frac{sinalpha }{cosalpha })

У меня же в выражении есть просто косинусы и синусы. Что нам нужно сделать?

Давай возьмем и «насильно» разделим числитель и знаменатель дроби на ( displaystyle cosalpha ). Это поможет мне «выделить» тангенс в чистом виде:

( displaystyle frac{10cosalpha +4sinalpha +15}{2sinalpha +5cosalpha +3}=frac{frac{10cosalpha +4sinalpha +15}{cosalpha }}{frac{2sinalpha +5cosalpha +3}{cosalpha }}=frac{10+4tgalpha +frac{15}{cosalpha }}{2tgalpha +5+frac{3}{cosalpha }}).

Конечно, есть одна неприятность: у нас появились дроби с косинусами. Но есть надежда, что мы с ними справимся! А пока что давай подставим вместо ( displaystyle tga) его числовое значение ( displaystyle -2,5). Тогда получим:

9. Нужно найти ( displaystyle 7cos left( pi +beta right)-2text{sin}left( frac{pi }{2}+beta right)), если дано ( displaystyle cosbeta =-frac{1}{3}).

Давай опять проанализируем, что нам нужно вычислить: искомая формула состоит из разности косинуса от суммы двух углов и синуса от суммы двух углов.

Давай упрощать: раскроем каждую из сумм (опять-таки повторюсь, что далее я опишу способ, который позволит обходиться без раскрытия такого рода сумм):

( displaystyle cos left( pi +beta right)=cospi cdot cosbeta -sinpi cdot sinbeta )

Опять-таки, тебе должно быть известно, что ( displaystyle cospi =-1,~~sinpi =0).

Если тебе это неизвестно, то настоятельно рекомендую тебе повторить тему тригонометрическая окружность.

Тогда моя формула примет вид:

( displaystyle cos left( pi +beta right)=-cosbeta =-left( -frac{1}{3} right)=frac{1}{3})

Теперь с синусом:

Формулы приведения

Теперь мы знаем уже почти что все. Осталось совсем немного. Последнее, на что я хочу обратить внимание, это обещанный мною метод «легкого» перехода от большой таблицы значений углов к маленькой.

Этот переход обеспечивают так называемые формулы приведения. Еще раз поясню, зачем они используются: ты будешь их применять в том случае, когда тебе нужно найти синус, косинус или тангенс угла, большего чем ( displaystyle 90) градусов.

Например, найти синус угла ( displaystyle 855) градусов.

Здесь мы поступаем следующим образом. Во-первых, нам понадобятся следующие знания:

Синус и косинус имеют период ( displaystyle 2pi ) (( displaystyle 360) градусов), то есть ( displaystyle sinleft( 2pi k+x right)=sinx)
( displaystyle cosleft( 2pi k+x right)=cosx)
Тангенс (котангенс) имеют период ( displaystyle pi ) (( displaystyle 180) градусов) ( displaystyle tgleft( pi k+x right)=tgx)
( displaystyle ctgleft( pi k+x right)=ctgx)
( displaystyle k) – любое целое число
Синус и тангенс – функции нечетные, а косинус – четная: ( displaystyle sinleft( -x right)=-sinx)
( displaystyle tgleft( -x right)=-tgleft( x right))
( displaystyle cosleft( -x right)=cosleft( x right))

Алгоритм использования формул приведения

Шаг 1. Если мы вычисляем значение тригонометрической функции от отрицательного угла – делаем его положительным при помощи группы формул (2).

Например:

( displaystyle sinleft( -855{}^circ right)=-sin855{}^circ ,~cosleft( -855{}^circ right)=cos855{}^circ )

Шаг 2. Отбрасываем для синуса и косинуса его периоды: ( displaystyle 2pi k) (по ( displaystyle 360) градусов), а для тангенса – «половинки» ( displaystyle pi k) (( displaystyle 180) градусов).

Например:

( displaystyle sin 855{}^circ =sinleft( 2cdot 360{}^circ +135{}^circ right)=sin 135{}^circ ) 

( displaystyle tg 225{}^circ =tgleft( 180{}^circ +45{}^circ right)=tg 45{}^circ )

Шаг 3. Если оставшийся «уголок» меньше ( displaystyle 90) градусов, то задача решена: ищем его в «малой таблице»

Шаг 4. Иначе ищем, в какой четверти лежит наш угол ( displaystyle alpha ): это будет 2, 3 или 4 четверть. Смотрим, какой знак имеет искомая функция в четверти. Запомнили этот знак!!!

Шаг 5. Представляем угол ( displaystyle alpha ) в одной из следующих форм:

  • ( displaystyle alpha =90+beta ) (если во второй четверти),
  • ( displaystyle alpha =180-beta ) (если во второй четверти),
  • ( displaystyle alpha =180+beta ) (если в третьей четверти),
  • ( displaystyle alpha =270-beta ) (если в третьей четверти),
  • ( displaystyle alpha =270+beta ) (если в четвертой четверти),
  • ( displaystyle alpha =360-beta ) (если в четвертой четверти).

…так, чтобы оставшийся угол ( displaystyle beta ) был больше нуля и меньше ( displaystyle 90) градусов.

Например: ( displaystyle 135{}^circ =180{}^circ -45{}^circ )

( displaystyle 135{}^circ =90{}^circ +45{}^circ )

( displaystyle 315{}^circ =270{}^circ+45{}^circ )

( displaystyle 240{}^circ =180{}^circ +60{}^circ )

( displaystyle 240{}^circ =270{}^circ -30{}^circ )…

В принципе не важно, в какой из двух альтернативных форм для каждой четверти ты представишь угол. На конечном результате это не скажется.

Шаг 6. Теперь смотрим, что у нас получилось:  

  • если ты выбрал запись через ( displaystyle 180) или ( displaystyle 360) градусов плюс минус что-либо, то знак функции меняться не будет: ты просто убираешь ( displaystyle 180) или ( displaystyle 360) и записываешь синус, косинус или тангенс оставшегося угла.
  • eсли же ты выбрал запись через ( displaystyle 90) или ( displaystyle 270) градусов, то синус меняем на косинус, косинус на синус, тангенс на котангенс, котангенс – на тангенс.

Шаг 7. Ставим перед получившимся выражением знак из пункта 4.

3 примера на тренировку

  • Вычислить ( displaystyle sin 2130{}^circ )
  • Вычислить ( displaystyle sqrt{2}cosfrac{21pi }{4})
  • Най­ди­те зна­че­ние вы­ра­же­ния: ( displaystyle 12sin 150{}^circ cos 120{}^circ )

Решения:

1. ( displaystyle sin 2130{}^circ )

Действуем согласно нашему алгоритму. Выделяем целое число кругов для ( displaystyle 2130{}^circ ):

( displaystyle frac{2130{}^circ }{360{}^circ }=5,91ldots )

В общем, делаем вывод, что в угол ( displaystyle 2130{}^circ ) помещается целиком 5 раз по ( displaystyle 360{}^circ ), а сколько осталось? Осталось ( displaystyle 2130{}^circ -5cdot 360{}^circ =330{}^circ ). Тогда:

( displaystyle sin~ 2130{}^circ =sinleft( 5cdot 360{}^circ +330{}^circ right)=sin 330{}^circ )

Ну вот, лишнее мы отбросили. Теперь разбираемся со знаком.

( displaystyle 330{}^circ ) лежит в 4 четверти. Синус четвертой четверти имеет знак «минус», его я и не должен забыть поставить в ответе. Далее, представляем ( displaystyle 330{}^circ ) согласно одной из двух формул пункта 5 правил приведения. Я выберу: ( displaystyle 330{}^circ =270{}^circ +60{}^circ )

( displaystyle sin 330{}^circ =sinleft( 270{}^circ +60{}^circ right))

Теперь смотрим, что получилось: у нас случай с ( displaystyle 270) градусами, тогда отбрасываем ( displaystyle 270{}^circ ) и синус меняем на косинус. И ставим перед ним знак «минус»!

( displaystyle sinleft( 270{}^circ +60{}^circ right)=-cos60{}^circ )

( displaystyle 60) градусов – угол в первой четверти. Мы знаем (ты мне обещал выучить малую таблицу!) его значение:

( displaystyle cos 60{}^circ =0,5)

Тогда получим окончательный ответ:

( displaystyle sin~ 2130{}^circ =-0,5)

Ответ: ( displaystyle -0,5)

2. ( displaystyle sqrt{2}cosfrac{21pi }{4}) 

Все то же самое, но вместо градусов – радианы. Ничего страшного. Главное помнить, что

( displaystyle pi ~рад.=180{}^circ )

Но можно и не заменять радианы на градусы. Это вопрос твоего вкуса. Я не буду ничего менять. Начну опять-таки с отбрасывания целых кругов:

( displaystyle frac{21pi }{4}=5frac{1}{4}pi =4pi +1frac{1}{4}pi )

Отбрасываем ( displaystyle 4pi ) – это два целых круга. Осталось вычислить ( displaystyle cos 1frac{1}{4}pi ). 

3. ( displaystyle 12sin 150{}^circ cos 120{}^circ ).

Нужно проделать все то же самое, но уже с двумя функциями.

Я буду несколько более краток: ( displaystyle 150{}^circ ) и ( displaystyle 120{}^circ ) градусов – углы второй четверти. Косинус второй четверти имеет знак «минус», а синус – «плюс».

( displaystyle 150{}^circ ) можно представить как: ( displaystyle 150{}^circ =90{}^circ +60{}^circ ), а ( displaystyle 120{}^circ ) как ( displaystyle 90{}^circ +30{}^circ ), тогда:

10 примеров на тренировку

Реши эти 10 заданий, и ты научишься пользоваться формулами тригонометрии!

Ну вот, теперь на мой взгляд, ты готов к решению всех оставшихся «за бортом» задач. Страшные углы теперь тебе более не помеха. Попробуй прорешать примеры самостоятельно, а потом мы с тобой сравним результаты.

  • ( displaystyle frac{5cos29{}^circ }{sin61{}^circ })
  • ( displaystyle frac{8}{sin left( -frac{27pi }{4} right)text{cos}left( frac{31pi }{4} right)})
  • ( displaystyle -4sqrt{3}text{cos}left( -750{}^circ right))
  • ( displaystyle 2sqrt{3}tgleft( -300{}^circ right))
  • ( displaystyle frac{14sin409{}^circ }{sin49{}^circ })
  • ( displaystyle frac{12}{si{{n}^{2}}27{}^circ +co{{s}^{2}}207{}^circ })
  • ( displaystyle frac{5sin74{}^circ }{cos37{}^circ cos53{}^circ })
  • ( displaystyle sqrt{3}co{{s}^{2}}frac{5pi }{12}-sqrt{3}si{{n}^{2}}frac{5pi }{12})
  • Най­ди­те зна­че­ние вы­ра­же­ния ( displaystyle 5tgleft( 5pi -gamma right)-tgleft( -gamma right)), если ( displaystyle tggamma =7).
  • Най­ди­те ( displaystyle sin left( frac{7pi }{2}-alpha right)), если ( displaystyle sinalpha =0,8) и ( displaystyle alpha in left( frac{pi }{2};pi right)).

Решения:

1. ( displaystyle frac{5cos29{}^circ }{sin61{}^circ }) 

Ключ к успеху – заметить, что:

( displaystyle 29{}^circ +61{}^circ =90{}^circ )!!! 

Тогда, например ( displaystyle 90{}^circ -61{}^circ =29{}^circ ):

( displaystyle frac{5cos29{}^circ }{sin61{}^circ }=frac{5text{cos}left( 90{}^circ -61{}^circ right)}{sin61{}^circ })

( displaystyle 90{}^circ -61{}^circ )– угол первой четверти. Косинус первой четверти – положительный. Поскольку мы вычитаем из ( displaystyle 90) градусов, то косинус меняется на синус:

( displaystyle frac{5text{cos}left( 90{}^circ -61{}^circ right)}{sin61{}^circ }=frac{5sin61{}^circ }{sin61{}^circ }=5)

Ответ: ( displaystyle 5).

2. ( displaystyle frac{8}{sin left( -frac{27pi }{4} right)text{cos}left( frac{31pi }{4} right)})

( displaystyle frac{8}{sin left( -frac{27pi }{4} right)text{cos}left( frac{31pi }{4} right)})

Опять задача целиком на формулы приведения. Вначале….

( displaystyle frac{8}{sin left( -frac{27pi }{4} right)text{cos}left( frac{31pi }{4} right)}=frac{8}{-frac{sqrt{2}}{2}cdot frac{sqrt{2}}{2}}=frac{8}{-frac{2}{4}}=-8:left( frac{2}{4} right)=-16)

…избавимся от минуса, вынеся его перед синусом (поскольку синус – функция нечетная!!!). Затем рассмотрим углы:

( displaystyle frac{27pi }{4}=frac{26pi }{4}+frac{pi }{4}=6pi +frac{pi }{4})

Отбрасываем целое количество кругов – то есть три круга (( displaystyle 6pi )). Остается вычислить: ( displaystyle sinfrac{pi }{4}=frac{sqrt{2}}{2})

Так же поступаем и со вторым углом:

( displaystyle frac{31pi }{4}=7frac{3}{4}pi =7pi +frac{3}{4}pi )

Удаляем целое число кругов –3 круга (( displaystyle 6pi )) тогда:

( displaystyle text{cos}left( frac{31pi }{4} right)=cos left( 7pi +frac{3}{4}pi right)=cos left( pi +frac{3}{4}pi right))

Теперь думаем: в какой четверти лежит оставшийся угол?

3. ( displaystyle -4sqrt{3}text{cos}left( -750{}^circ right))

Стандартно: убираем минус из косинуса, пользуясь тем, что ( displaystyle cosleft( -x right)=cosleft( x right)).

Осталось сосчитать косинус ( displaystyle 750) градусов. Уберем целые круги: ( displaystyle 750{}^circ =2cdot 360{}^circ +30{}^circ ).

Тогда:

4. ( displaystyle 2sqrt{3}tgleft( -300{}^circ right))

( displaystyle 2sqrt{3}tgleft( -300{}^circ right))

Действуем так же, как в предыдущем примере.

( displaystyle tgleft( -300{}^circ right)=-tg300{}^circ )

Поскольку ты помнишь, что период у тангенса – ( displaystyle 180) градусов (или ( displaystyle pi )) в отличие от косинуса или синуса, у которых он в 2 раза больше, то удалим целое количество ( displaystyle pi ).

5. ( displaystyle frac{14sin409{}^circ }{sin49{}^circ })

Снизу у нас все хорошо – маленький уголок первой четверти. Наверху же – все плохо.

Угол большой, надо его упростить по формулам приведения:

6. ( displaystyle frac{12}{si{{n}^{2}}27{}^circ +co{{s}^{2}}207{}^circ })

Вся проблема, как ты понимаешь, в косинусе. Но не беда, решим.

Смотри, на знак нам все равно, поскольку косинус-то у нас в квадрате и знак всегда будет «плюс».То есть на четверти можно не смотреть.

В то же время:

7. ( displaystyle frac{5sin74{}^circ }{cos37{}^circ cos53{}^circ })

Пример немного похитрее. Прежде всего заметим, что ( displaystyle 74{}^circ =2cdot 37{}^circ ). Тогда давай представим числитель как синус двойного угла!

( displaystyle frac{5sin74{}^circ }{cos37{}^circ cos53{}^circ }=frac{5cdot 2sin37{}^circ cos37{}^circ }{cos37{}^circ cos53{}^circ }=frac{10sin37{}^circ }{cos53{}^circ })

Тебе это ничего не напоминает? Задача в точности такая же, как в номере 1. Я тогда так и поступлю, заметив, что у меня опять:

8. ( displaystyle sqrt{3}co{{s}^{2}}frac{5pi }{12}-sqrt{3}si{{n}^{2}}frac{5pi }{12})

Опять задание комбинированное! Легко увидеть и вынести за скобки общий множитель ( displaystyle sqrt{3}):

( displaystyle sqrt{3}co{{s}^{2}}frac{5pi }{12}-sqrt{3}si{{n}^{2}}frac{5pi }{12}=sqrt{3}left( co{{s}^{2}}frac{5pi }{12}-si{{n}^{2}}frac{5pi }{12} right))

Как называется формула внутри скобок? Пробегись глазами по списку наших формул! Нашел? Это косинус двойного угла!

9. Най­ди­те зна­че­ние вы­ра­же­ния ( displaystyle 5tgleft( 5pi -gamma right)-tgleft( -gamma right)), если ( displaystyle tggamma =7).

У тангенса период – ( displaystyle pi ), так что не задумываясь отбрасываем его:

( displaystyle 5tgleft( 5pi -gamma right)=5tgleft( -gamma right) =-5tggamma )

Здесь мы использовали еще и тот факт, что тангенс – функция нечетная.

10. Най­ди­те ( displaystyle sin left( frac{7pi }{2}-alpha right)), если ( displaystyle sinalpha =0,8) и ( displaystyle alpha in left( frac{pi }{2};pi right))

Вначале упростим выражение, используя формулы приведения (вначале отбросим целые круги и уберем минус):

( displaystyle sin left( frac{7pi }{2}-alpha right)=sin left( 2pi -frac{pi }{2}-alpha right)=sin left( -frac{pi }{2}-alpha right)=-text{sin}left( frac{pi }{2}+alpha right))

Наш оставшийся угол – во третьей четверти (посмотри на условия для угла в условии задачи!!!).

Средний уровень сложности

В некоторых (не очень тривиальных) случаях, следующие формулы помогут тебе выйти из затруднительной ситуации.

Первая группа формул является универсальной: она позволяет перейти от любого тригонометрического выражения к рациональному.

Это, конечно, имеет важное приложение при решении уравнений, но здесь мы рассмотрим, как эти формулы помогают при упрощении тригонометрических выражений.

Формулы понижения степени

  • ( displaystyle {sin^{2}}alpha =frac{1-cos2alpha }{2})
  • ( displaystyle {cos^{2}}alpha =frac{1+cos2alpha }{2})
  • ( displaystyle t{{g}^{2}}alpha =frac{1-cos2alpha }{1+cos2alpha },alpha ne frac{pi }{2}+pi n,nin Z)

Универсальная тригонометрическая подстановка

  • ( displaystyle sinalpha =frac{2tgfrac{alpha }{2}}{1+t{{g}^{2}}frac{alpha }{2}})
  • ( displaystyle cosalpha =frac{1-t{{g}^{2}}frac{alpha }{2}}{1+t{{g}^{2}}frac{alpha }{2}})
  • ( displaystyle tgalpha =frac{2tgfrac{alpha }{2}}{1-t{{g}^{2}}frac{alpha }{2}})
  • ( displaystyle ctgalpha =frac{1-t{{g}^{2}}frac{alpha }{2}}{2tgfrac{alpha }{2}})

В чем прелесть этих формул? Первые две позволяют «убрать степени», то есть понизить порядок выражения (или повысить, за счёт снижения кратности угла), вторая группа формул позволяет свести любое тригонометрическое выражение к виду, зависящему только от тангенсов!

Иногда это единственный способ решить ту или иную задачу.

Разбор 3 примеров

1. Доказать тождество: ( displaystyle frac{3-4cos2alpha +cos4alpha }{3+4cos2alpha +cos4alpha }=t{{g}^{4}}alpha )

С виду тождество угрожающе! Но разберёмся по порядку. Формулы понижения степени, конечно, если их прочитать задом наперёд повышают степень!

И вообще, приглядись внимательно: первые две формулы есть ничто иное, как косинус двойного угла, записанный в несколько странной форме!

Вот и распишем по правилам:

( displaystyle begin{array}{l}frac{3-4cos2alpha +cos4alpha }{3+4cos2alpha +cos4alpha }=frac{3-4cos2alpha +left( 2{cos^{2}}2alpha -1 right)}{3+4cos2alpha +left( 2{cos^{2}}2alpha -1 right)}=\=frac{2-4cos2alpha +2{cos^{2}}2alpha }{2+4cos2alpha +2{cos^{2}}2alpha }=frac{1-2cos2alpha +{cos^{2}}2alpha }{1+2cos2alpha +{cos^{2}}2alpha }end{array})

Тебе ничего по форме не напоминают числитель и знаменатель дроби? Приглядись внимательно, здесь «зарыта» хорошо известная тебе формула. Увидел её? Это же квадрат разности и квадрат суммы! (Подробнее об этом читай в статье о  формулах сокращенного умножения)

( displaystyle frac{1-2cos2alpha +{cos^{2}}2alpha }{1+2cos2alpha +{cos^{2}}2alpha }=frac{{{left( 1-cos2alpha right)}^{2}}}{{{left( 1+cos2alpha right)}^{2}}}={{left( frac{1-cos2alpha }{1+cos2alpha } right)}^{2}})

А выражение в скобках есть ничто иное, как ( displaystyle t{{g}^{2}}alpha ), окончательно получим:

( displaystyle {{left( frac{1-cos2alpha }{1+cos2alpha } right)}^{2}}={{left( t{{g}^{2}}alpha right)}^{2}}=t{{g}^{4}}alpha )

Тождество доказано!

Следующий пример очень схож с предыдущим, постарайся решить его самостоятельно.

2. Доказать тождество: ( displaystyle frac{1+sin2alpha +cos2alpha }{1+sin2alpha -cos2alpha }=ctgalpha )

Решение (хотя может и отличаться от твоего):

Опять «повысим степень» у косинуса: ( displaystyle cos2alpha =2{cos^{2}}alpha -1)

( displaystyle frac{1+sin2alpha +cos2alpha }{1+sin2alpha -cos2alpha }=frac{1+sin2alpha +2{cos^{2}}alpha -1}{1+sin2alpha -2{cos^{2}}alpha +1}=frac{sin2alpha +2{cos^{2}}alpha }{2+sin2alpha -2{cos^{2}}alpha })

Надо сокращать дальше! Что делать? Ясно, что надо избавляться от двойных углов у синуса. Действуем по формуле синуса двойного угла и сокращаем двойки:

( displaystyle frac{sin2alpha +2{cos^{2}}alpha }{2+sin2alpha -2{cos^{2}}alpha }=frac{2sin{alpha} cos{alpha} +2{cos^{2}}alpha }{2+2sin{alpha} cos{alpha}-2{cos^{2}}alpha }=frac{sinalpha cosalpha +{cos^{2}}alpha }{1+sinalpha cos{alpha}-{cos^{2}}alpha })

Числитель раскладывается на множители. Знаменатель –пока нет. До тех пор, пока мы не применим основное тригонометрическое тождество:

( displaystyle 1-{cos^{2}}alpha ={sin^{2}}alpha )

( displaystyle frac{sinalpha cosalpha +{cos^{2}}alpha }{1+sinalpha cosalpha -{cos^{2}}alpha }=frac{sinalpha cosalpha +{cos^{2}}alpha }{{sin^{2}}alpha +sinalpha cosalpha }=frac{cosalpha left( sinalpha +cosalpha right)}{sinalpha left( sinalpha +cosalpha right)}=ctgalpha )

Вот ещё один пример, но не такой простой.

3. Доказать, что если ( displaystyle 0<alpha <frac{pi }{2}), то ( displaystyle sqrt{1+sinalpha }-sqrt{1-sinalpha }=2sinfrac{alpha }{2})

Зачем нам дан угол? Наверное, чтобы оценить выражения: синус ( displaystyle alpha )будет положительным, ( displaystyle sinfrac{alpha }{2}>0,~1+sinalpha >1,~0<1-sinalpha <1)

Тогда и левая, и правая части тождества больше нуля. Это даёт мне право без задней мысли возвести их в квадрат:

( displaystyle {{left( sqrt{1+sinalpha }-sqrt{1-sinalpha } right)}^{2}}=4{sin^{2}}frac{alpha }{2}) – вот такое тождество нам нужно теперь доказать.

Раскроем скобки в левой части по формуле квадрата разности!

( displaystyle begin{array}{l}{{left( sqrt{1+sin alpha }-sqrt{1-sin alpha } right)}^{2}}=1+sin alpha -2sqrt{1+sin alpha }cdot sqrt{1-sin alpha }+1-\-sin alpha =2-2sqrt{1+sin alpha }cdot sqrt{1-sin alpha }=2left( 1-sqrt{1+sin alpha }cdot sqrt{1-sin alpha } right)=\2left( 1-sqrt{1+{{sin }^{2}}alpha } right)=2left( 1-sqrt{{cos^{2}}}alpha right)end{array})

Я не сомневаюсь в твоей грамотности и поэтому даже не упоминаю про использованные мною формулы в выкладках. 

Теперь надо бы убрать корень из косинуса. Но мы знаем, что просто так это делать нельзя, ибо ( displaystyle sqrt{{{a}^{2}}}=left| a right|). 

В то же время вспоминаем про четверть: наш угол лежит в первой четверти, тогда косинус имеет знак «плюс» и мы просто убираем корень: 

( displaystyle 2left( 1-sqrt{{cos^{2}}}alpha right)=2left( 1-cosalpha right))

Тогда нам надо доказать, что

( displaystyle 2left( 1-cosalpha right)=4{sin^{2}}frac{alpha }{2})

( displaystyle left( 1-cosalpha right)=2{sin^{2}}frac{alpha }{2})

Справа применим формулу понижения степени:

( displaystyle {sin^{2}}frac{alpha }{2}=frac{1-cosalpha }{2}), тогда ( displaystyle 2{sin^{2}}frac{alpha }{2}=1-cosalpha )

Тождество доказано!

Конечно, можно привести ещё массу примеров, где применяются формулы понижения степени, ты их и сам без труда отыщешь.

Теперь вторая (и заключительная в этом обзоре) группа формул – формулы преобразования произведения в сумму и суммы в произведение.

Формулы преобразования суммы функций

  • ( displaystyle sinalpha pm sinbeta =2sinfrac{alpha pm beta }{2}cosfrac{alpha mp beta }{2})
  • ( displaystyle cosalpha +cosbeta =2cosfrac{alpha +beta }{2}cosfrac{alpha -beta }{2})
  • ( displaystyle cosalpha -cosbeta =-2sinfrac{alpha +beta }{2}sinfrac{alpha -beta }{2})
  • ( displaystyle tgalpha pm tgbeta =frac{sinleft( alpha pm beta right)}{cosalpha cosbeta })
  • ( displaystyle ctgalpha pm ctgbeta =frac{sinleft( beta pm alpha right)}{sinalpha sinbeta })

Иногда бывают полезны и обратные преобразования.

Формулы преобразования произведений функций

  • ( displaystyle sinalpha sinbeta =frac{cos left( alpha -beta right)-cosleft( alpha +beta right)}{2})
  • ( displaystyle sinalpha cosbeta =frac{sin left( alpha +beta right)+sinleft( alpha -beta right)}{2})
  • ( displaystyle cosalpha cosbeta =frac{cos left( alpha -beta right)+cosleft( alpha +beta right)}{2})

Решение 5 примеров

1. Доказать тождество: ( displaystyle frac{sinalpha +sin3alpha }{cosalpha +cos3alpha }=tg2alpha )

Давай не будем долго думать, а, как говорится, пойдём в лобовую атаку: в числителе и знаменателе перейдём от суммы к произведению:

( displaystyle begin{array}{l}~frac{sinalpha+sin3alpha}{cosalpha+cos3alpha}=frac{2sinfrac{alpha+3alpha}{2}cosfrac{alpha-3alpha}{2}}{2cosfrac{alpha+3alpha}{2}cosfrac{alpha-3alpha}{2}}=frac{2cdot sin2alphacdot cosleft( -alpha right)}{2cdot cos2alphacdot cosleft( -alpha right)}=\=frac{sin2alpha}{cos2alpha}=tg2alphaend{array})

И минуты не прошло, а пример уже решён!

Теперь попробуй сам.

2. Доказать тождество: ( displaystyle frac{sin2alpha +sin4alpha }{cos2alpha -cos4alpha }=ctgalpha )

Решение – опять лобовая атака:

( displaystyle begin{array}{l}frac{sin2alpha+sin4alpha}{cos2alpha-cos4alpha}=frac{2sinfrac{2alpha+4alpha}{2}cosfrac{2alpha-4alpha}{2}}{-2sinfrac{2alpha+4alpha}{2}sinfrac{2alpha-4alpha}{2}}=frac{2sin3alphacdot cosleft( -alpha right)}{-2sin3alphacdot sinleft( -alpha right)}=frac{cosleft( -alpha right)}{-sinleft( -alpha right)}end{array})

Так как синус – функция нечётная, а косинус – чётная, то:

( displaystyle frac{cosleft( -alpha right)}{-sinleft( -alpha right)}=frac{cosalpha }{-left( -sinalpha right)}=frac{cosalpha }{sinalpha }=ctgalpha )

Этот пример чуть похитрее, будь внимателен!

3. Доказать тождество: ( displaystyle frac{sin2alpha +sin5alpha -sin3alpha }{cosalpha +1-2{sin^{2}}2alpha }=2sinalpha )

Я не хочу трогать синус двойного угла. Уж больно он удобно раскладывается на множители, чего не скажешь о синусе тройного и тем более пятикратного угла.

Поэтому я сверну в произведение последние 2 слагаемых в числителе:

( displaystyle begin{array}{l}frac{sin2alpha +sin5alpha -sin3alpha }{cosalpha +1-2{sin^{2}}2alpha }=frac{sin2alpha +2sinfrac{5alpha -3alpha }{2}cosfrac{5alpha +3alpha }{2}}{cosalpha +1-2{sin^{2}}2alpha }=\=frac{2sinalpha cosalpha +2sinalpha cos4alpha }{cosalpha +1-2{sin^{2}}2alpha }=frac{2sinalpha left( cosalpha +cos4alpha right)}{cosalpha +1-2{sin^{2}}2alpha }end{array})

Конечно, теперь можно было бы и свернуть числитель ещё раз, но я пойду иным путём. В знаменателе у меня тоже спрятана формула, вот она: 

( displaystyle 1-2{sin^{2}}2alpha ). 

Что это за формула? Это косинус двойного угла!

( displaystyle 1-2{sin^{2}}2alpha =cosleft( 2cdot 2alpha right)=cos4alpha )

( displaystyle frac{2sinalpha left( cosalpha +cos4alpha right)}{cosalpha +1-2{sin^{2}}2alpha }=frac{2sinalpha left( cosalpha +cos4alpha right)}{cosalpha +cos4alpha }=2sinalpha )

Тождество доказано!

Теперь попробуй решить вот этот пример для закрепления пройденного материала.

4. Доказать тождество: ( displaystyle {cos^{4}}alpha -{sin^{4}}alpha +sin2alpha =sqrt{2}cosleft( 2alpha -frac{pi }{4} right))

Проверяем!

( displaystyle begin{array}{l}{cos^{4}}alpha -{sin^{4}}alpha +sin2alpha =left( {cos^{2}}alpha -{sin^{2}}alpha right)left( {cos^{2}}alpha +{sin^{2}}alpha right)+sin2alpha =\=cos2alpha +sin2alpha end{array})

C другой стороны:

( displaystyle begin{array}{l}sqrt{2}cos left( 2alpha-frac{pi }{4} right)=sqrt{2}left( cos{2alpha}cos{frac{pi }{4}}+sin{2alpha}sin{frac{pi }{4}} right)=\=sqrt{2}left( frac{sqrt{2}}{2}cos2alpha+frac{sqrt{2}}{2}sin2alpha right)=sqrt{2}cdot frac{sqrt{2}}{2}left( cos2alpha+sin2alpha right)=\=cos2alpha+sin2alphaend{array})

Тождество доказано!

На этом примере я буду закругляться потихоньку.

Сразу оговорюсь: не переживай и не волнуйся, если у тебя что-то сразу не выходит. Тригонометрия – сложная и очень обширная тема. Здесь все зависит не только от знания формул, но и от мастерства и смекалки. На их выработку тебе понадобится время и усердие.

Более того, скажу тебе вот что: изначально я хотел вставить другой пример в качестве заключительного. Однако на его решение мне понадобилось около 20 минут, причём я использовал ещё более сложную методику его решения. Так что не только ты сталкиваешься с трудностями при решении примеров, трудности бывают у всех! 

Все-таки я приведу здесь этот трудный пример, вдруг да и получится у тебя решить его, может, я что-то упустил. Вот он:

5. Упростить: ( displaystyle frac{1+sinalpha -cos2alpha -sin3alpha }{2{sin^{2}}alpha +sinalpha -1})

А вот какой у меня получился в итоге ответ: ( displaystyle 2sinalpha.)

Дерзай!

В следующей части статьи я рассмотрю его решение, но прибегну к ещё более изощрённой технике нежели та, что рассматривалась здесь! Удачи!

Повышенный уровень сложности

В дополнение к уже изложенному материалу, я бы хотел рассмотреть еще небольшую группку формул, которая осталась «за бортом».

Эти формулы – некоторое обобщение уже рассмотренных ранее формул понижения степени, но вот понижаемые степени у них повыше.

Формулы понижения 3-й степени

  • ( displaystyle si{{n}^{3}}alpha =frac{3sinalpha -sin3alpha }{4})
  • ( displaystyle co{{s}^{3}}a=frac{3cosa+cos3a}{4})

Из данных формул можно вывести формулы тройного угла.

Формулы тройного угла

  • ( displaystyle sin3alpha =3sinalpha -4si{{n}^{3}}alpha )
  • ( displaystyle cos3a=4co{{s}^{3}}a-3cosa)
  • ( displaystyle tg3alpha =frac{3tgalpha -t{{g}^{3}}alpha }{1-3t{{g}^{2}}alpha })
  • ( displaystyle ctg3alpha =frac{3ctgalpha -ct{{g}^{3}}alpha }{1-3ct{{g}^{2}}alpha })

Ты мне можешь задать резонный вопрос: как часто эти формулы используются? Я отвечу: постарайся избегать прибегать к ним. Они нужны на тот случай, когда ничего другого уже не можешь придумать.

В частности, они могут быть полезными при решении сложных уравнений, которые встречаются во вступительных экзаменах на математические специальности. 

Однако уравнениям у нас будет посвящена отдельная статья, так что здесь я рассмотрю случаи, когда данные формулы позволяют упрощать тригонометрические выражения.

Пример 1

Упростить: ( displaystyle A=frac{1}{3}co{{s}^{3}}alpha cdot sin3alpha +frac{1}{3}si{{n}^{3}}alpha cdot cos3alpha )

Решение:

Подставим вместо ( displaystyle sin3alpha ) и ( displaystyle cos3alpha ) их представления согласно формулам тройного угла, тогда:

( displaystyle begin{array}{l}A=frac{1}{3}co{{s}^{3}}alpha left( 3sinalpha -4si{{n}^{3}}alpha right)+frac{1}{3}si{{n}^{3}}alpha left( 4co{{s}^{3}}alpha -3cosalpha right)=\=co{{s}^{3}}alpha cdot sinalpha -frac{4}{3}co{{s}^{3}}alpha cdot si{{n}^{3}}alpha +frac{4}{3}co{{s}^{3}}alpha cdot si{{n}^{3}}alpha -si{{n}^{3}}alpha cdot cosalpha =\=co{{s}^{3}}alpha cdot sinalpha -si{{n}^{3}}alpha cdot cosalpha end{array})

Теперь вынесем в оставшемся выражении общий множитель за скобки:

( displaystyle co{{s}^{3}}alpha cdot sinalpha -si{{n}^{3}}alpha cdot cosalpha =sinalpha cdot cosalpha left( co{{s}^{2}}alpha -si{{n}^{2}}alpha right))

По формулам двойного угла: ( displaystyle sinalpha cdot cosalpha =frac{1}{2}sin2alpha ), ( displaystyle co{{s}^{2}}alpha -si{{n}^{2}}alpha =cos2alpha ):

( displaystyle sinalpha cdot cosalpha left( co{{s}^{2}}alpha -si{{n}^{2}}alpha right)=frac{1}{2}sin2alpha cdot cos2alpha )

Ну а здесь снова спрятан синус двойного угла:

( displaystyle frac{1}{2}sin2alpha cdot cos2alpha =frac{1}{4}sin4alpha )

Ответ: ( displaystyle A=frac{1}{4}sin4alpha )

Следующий пример попробуй решить самостоятельно. Не уверен, что в нем обязательно использовать формулу тройного угла, но можно сделать и с ее помощью.

Пример 2

Упростить: ( displaystyle frac{1+sinalpha -cos^2{alpha}-cos2alpha -sin3alpha }{2si{{n}^{2}}alpha +sinalpha -1})

Решение:

Моя цель – свести числитель дроби к выражению, зависящему только от синусов одиночного угла. Для этого я преобразую

( displaystyle cos^2 {alpha} =1-si{{n}^{2}}alpha )

( displaystyle cos2alpha =1-2si{{n}^{2}}alpha )

( displaystyle sin3alpha =3sinalpha -4si{{n}^{3}}alpha )

Имеем:

( displaystyle begin{array}{l}frac{1+sinalpha -cos2alpha -sin3alpha }{2si{{n}^{2}}alpha +sinalpha -1}=frac{1+sinalpha -left( 1-si{{n}^{2}}alpha right) -left( 1-2si{{n}^{2}}alpha right)-left( 3sinalpha -4si{{n}^{3}}alpha right)}{2si{{n}^{2}}alpha +sinalpha -1}=\=frac{4si{{n}^{3}}alpha +3si{{n}^{2}}alpha -2sinalpha -1}{2si{{n}^{2}}alpha +sinalpha -1}end{array})

Казалось бы, стало еще хуже. Но это так кажется. Давай для удобства вычислений заменим ( displaystyle sinalpha =t), тогда мне надо упростить дробь

( displaystyle frac{4{{t}^{3}}+3{{t}^{2}}-2t-1}{2{{t}^{2}}+t-1})

Нижнее выражение разложим на множители:

( displaystyle 2{{t}^{2}}+t-1=left( t+1 right)left( 2t-1 right))

С верхним фокус сложнее. Мы не умеем с тобой решать кубические уравнения. Но мы хорошо играем в «угадайку».

Угадай-ка один корень уравнения ( displaystyle 4{{t}^{3}}+3{{t}^{2}}-2t-1=0). Угадал? Я угадал ( displaystyle -1).

Тогда по теореме Безу (которую ты, быть может, знаешь, а если не знаешь, то без проблем отыщешь сам) выражение ( displaystyle 4{{t}^{3}}+3{{t}^{2}}-2t-1) делится без остатка на ( displaystyle t+1)

Разделим столбиком ( displaystyle 4{{t}^{3}}+3{{t}^{2}}-2t-1) на ( displaystyle t+1). Я получу:

( displaystyle 4{{t}^{3}}+3{{t}^{2}}-2t-1=left( t+1 right)left( 4{{t}^{2}}-t-1 right))

В свою очередь ( displaystyle 4{{t}^{2}}-t-1=4left( t-frac{1}{2} right)left( t+frac{1}{4} right))

Окончательно получим:

( displaystyle begin{array}{l}frac{4{{t}^{3}}+3{{t}^{2}}-2t-1}{2{{t}^{2}}+t-1}=frac{4left( t+1 right)left( t-frac{1}{2} right)left( t+frac{1}{4} right)}{left( t+1 right)left( 2t-1 right)}=frac{left( t+1 right)left( 2t-1 right)left( 2t+0,5 right)}{left( t+1 right)left( 2t-1 right)}=\=2t+0,5end{array})

Тогда исходное выражение можно упростить до: ( displaystyle 2sinx+0,5)

В завершение я приведу тебе пример одного уравнения, которое было предложено на психологический (???!!!) факультет одного из ВУЗов в 1990 году. Такие задачи называются задачи-гробы (никакая смекалка без знания конкретной формулы не позволит их решить):

Решить уравнение: ( displaystyle sqrt{3}co{{s}^{3}}x-3co{{s}^{2}}x-3sqrt{3}cosx+1=0)

Не сделав вот такую странную замену: ( displaystyle cosx=tgalpha ) решить его очень сложно. А с такой заменой у нас получится вот что:

( displaystyle sqrt{3}t{{g}^{3}}alpha -3t{{g}^{2}}alpha -3sqrt{3}tgalpha +1=0)

( displaystyle sqrt{3}t{{g}^{3}}alpha -3sqrt{3}tgalpha =3t{{g}^{2}}alpha -1)

( displaystyle sqrt{3}(t{{g}^{3}}alpha -3tgalpha )=3t{{g}^{2}}alpha -1)

( displaystyle -sqrt{3}left( 3tgalpha -t{{g}^{3}}alpha right)=-left( 1-3t{{g}^{2}}alpha right))

( displaystyle frac{left( 3tgalpha -t{{g}^{3}}alpha right)}{left( 1-3t{{g}^{2}}alpha right)}=frac{1}{sqrt{3}})

А вот ради чего весь этот сыр-бор: ( displaystyle frac{left( 3tgalpha -t{{g}^{3}}alpha right)}{left( 1-3t{{g}^{2}}alpha right)}=tg3alpha )

( displaystyle tg3alpha =frac{1}{sqrt{3}})

Это уравнение уже несказанно легче решается. Скоро мы вместе в этом убедимся. Но тут проблема в обратной замене… Тем не менее, эта задача почти нерешаема без знания формулы тангенса тройного угла. Вот так вот.

Бонусы: Вебинары из нашего курса подготовки к ЕГЭ по математике

ЕГЭ 9. Тригонометрическая окружность, табличные значения

На этом уроке мы узнаем, что такое тригонометрическая окружность и насколько она важна для тригонометрии. М

ы увидим, что она — основной инструмент в тригонометрии: с её помощью можно вывести любую формулу и найти любые значения.

Мы поймем, как «работает» окружность — а значит, поймём тригонометрию в целом.

ЕГЭ 13б. Тригонометрическая окружность

Тригонометрическая окружность — это очень простой и эффективный инструмент для решения любой тригонометрической задачи. На этом уроке вы узнаете как пользоваться тригонометрической окружностью для решения пункта «б» из задачи №13 профильного ЕГЭ.

Пункт “б” задачи №13 ЕГЭ 2020 В 2020 году на ЕГЭ в пункте «б» необходимо было указать корни тригонометрического уравнения принадлежащие отрезку.

Вообще-то решать пункт “б” можно двумя способами: — отметить корни уравнения на единичной окружности (способ разобранный в этом видео); — через двойное неравенство.

И вы должны знать, что второй способ чуть дольше, чем первый, но зато вы сможете проще описать все ваши рассуждения и вам будет сложнее ошибиться.

И еще один плюс второго способа — его проще оформить, так, чтобы к вам не придрались на ЕГЭ.

Мы считаем второй способ (через двойное неравенство) более предпочтительным на ЕГЭ по математике, но теме не менее для глубокого понимания темы (что может выручить на ЕГЭ) необходимо разобраться и с первым способом

Подготовка к ЕГЭ на 90+

Алексей Шевчук — ведущий мини-групп

математика, информатика, физика

+7 (905) 541-39-06 — WhatsApp/Телеграм для записи

alexei.shevchuk@youclever.org — email для записи

  • тысячи учеников, поступивших в лучшие ВУЗы страны
  • автор понятного всем учебника по математике ЮКлэва (с сотнями благодарных отзывов);
  • закончил МФТИ, преподавал на малом физтехе;
  • репетиторский стаж — c 2003 года;
  • в 2021 году сдал ЕГЭ (математика 100 баллов, физика 100 баллов, информатика 98 баллов — как обычно дурацкая ошибка:);
  • отзыв на Профи.ру: «Рейтинг: 4,87 из 5. Очень хвалят. Такую отметку получают опытные специалисты с лучшими отзывами».

Добавить комментарий